You are on page 1of 136

Cong ty CP T&PT Giao duc Phng Nam gi quyen cong bo tac pham.

Ma so : ...........M9
2

Muc luc
Thong tin quan trong ............................................................................................ 3
Thao tac mau ........................................................................................................ 3
Tr lai che o cai at ban au ............................................................................. 3
Than trong an toan ............................................................................................... 3
Than trong x l ..................................................................................................... 3
M nap may .......................................................................................................... 4
Bat va tat nguon ................................................................................................... 4
ieu chnh o tng phan hien th ....................................................................... 4
Nhan phm ............................................................................................................ 5
oc hien th ........................................................................................................... 5
Dung menu ........................................................................................................... 6
Xac nh phng thc tnh toan ........................................................................... 7
Cai at cho may.................................................................................................... 7
Khi au thiet b may tnh tay ............................................................................... 9
a vao bieu thc va gia tr ................................................................................. 9
Tnh toan thap phan tuan hoan .......................................................................... 12
Chuyen ket qua tnh toan ................................................................................... 16
Tnh toan c ban ................................................................................................. 17
Tnh ham ............................................................................................................. 21
Tnh toan thong ke (STAT) ................................................................................. 24
Tnh toan phng trnh (EQN) ............................................................................ 28
Tao ra bang so t ham (TABLE) ........................................................................ 30
Tnh toan bat phng trnh (INEQ) ..................................................................... 31
Tnh t so (RATIO) ............................................................................................... 34
Mien tnh toan, so ch so va o chnh xac ........................................................ 35
Loi ....................................................................................................................... 37
Trc khi xac nh may tnh tay truc trac... ........................................................ 38
Thay the pin ........................................................................................................ 39
ac ta ................................................................................................................. 39
Cau hoi thng gap ............................................................................................ 40
GIAI CAC BAI TOAN THUOC CHNG TRNH TRUNG HOC C S ........... 41
LP 6 .................................................................................................................. 41
LP 7 .................................................................................................................. 74
LP 8 ................................................................................................................ 102
ai so ........................................................................................................... 102
Hnh hoc ....................................................................................................... 110
LP 9 ................................................................................................................ 112
ai so ........................................................................................................... 112
Hnh hoc ....................................................................................................... 126

Thong tin quan trong

Cac hien th va minh hoa (nh cac nhan phm) c neu trong Tai
lieu hng dan ngi dung nay ch c dung vi muc ch minh
hoa, va co the khac vi cac khoan muc thc te chung bieu dien.

Hay gi tai lieu hng dan nay e tham khao ve sau.

Thao tac mau


Thao tac mau trong tai lieu nay c ch ra bi hnh tng
. Khi
khong c noi rieng, tat ca cac thao tac mau eu gia nh rang may
tnh tay c thiet at theo mac nh khi au. Hay dung thu tuc
muc Tr lai che o cai at ban au e a may tnh tay tr ve viec
thiet at mac nh ban au cua no.
e biet thong tin ve cac nhan
,
,
va
c ch ra
trong thao tac mau, xem Lap cau hnh thiet at may tnh tay.

Tr lai che o cai at ban au


Thc hien thu tuc sau khi ban muon tr lai che o cai at ban au va
tr ve phng thc tnh toan va thiet lap lai cac cai at mac nh ban
au. Lu y rang thao tac nay cung xoa i tat ca d lieu hien thi trong
bo nh may tnh tay.
(CLR)

(All)

(Yes)

Than trong an toan


Pin
e pin ngoai tam vi cua tre nho.
Ch dung kieu pin chuyen dung cho may tnh tay nay c
neu trong tai lieu nay.

Than trong x l

Cho du may tnh tay ang van hanh bnh thng, hay thay pin
t nhat mot lan trong vong hai nam.
Pin het co the do r, gay ra h hong va chay sai cho may tnh tay.
ng bao gi e pin het trong may tnh tay.

Pin i cung may tnh co xa ien trong qua trnh giao hang va
cat gi. Bi vay, co the can phai thay the pin nay sm hn tuoi
tho thng c trong i cho pin.
4

Khong dung pin oxyride hay bat k kieu pin chnh co nicken vi
san pham nay. S khong tng hp gia nhng pin o co the
lam giam tuoi tho cua pin va lam san pham van hanh truc trac.

Tranh dung va cat gi may tnh tay trong khu vc co nhiet o


qua cao hay qua thap hay moi trng am t va nhieu bui.

ng e may tnh tay b va cham, b e nen hay uon cong qua


mc.

ng bao gi th thao ri may tnh tay ra.

Dung vai mem, kho e lau ben ngoai may tnh tay.

Bat k khi nao vt bo may tnh tay hay pin, hay tuan theo luat
va quy nh cua khu vc ban .

Ten cong ti va san pham c dung trong tai lieu nay co the la cac
thng hieu a ang k hay cac thng hieu cua cac chu nhan
tng ng cua chung.

M nap may
Trc khi dung may tnh tay, hay trt nap
cng xuong e bo no ra, va roi gan nap
cng vao ang sau may tnh tay nh c
ve trong minh hoa ben canh.

Bat va tat nguon


Nhan
Nhan

e bat may tnh tay.


(OFF) e tat may tnh tay.

T ong tat nguon


May tnh tay cua ban se t ong tat nguon neu ban khong thc hien
thao tac nao trong khoang 10 phut. Neu ieu nay xay ra, nhan phm
e bat may tnh tay tr lai.

ieu chnh o tng phan hien th


Hien th man hnh CONTRAST bang viec thc hien thao tac sau
(SETUP)
( CONT ). Tiep o dung
va
e
ieu chnh o tng phan. Sau khi thiet at la ung ieu ban muon,
.
nhan
ieu quan trong: Neu ieu chnh tng phan hien th ma van khong
oc c, ieu o co the la nguon pin b yeu roi. Hay thay pin.
5

Nhan phm
Nhan phm
hay
tiep theo sau la
phm th hai se thc hien chc nang thay
phien cua phm th hai. Chc nang thay
phien c ch ra bi ch c in tren
phm nay.
Bang sau ch ra y ngha cac mau khac nhau
cua ch tren phm chc nang thay phien:

Chc nang thay phien

sin- 1 D

Chc nang in tren phm

Neu ch nhan cua


phm co mau:

Ngha la:

Vang

Nhan
roi nhan phm nay e truy nhap
vao ham ap dung c

roi nhan phm nay e a vao


Nhan
bien, hang hay k hieu ap dung c.

oc hien th
Hien th cua may tnh tay cho hien cac bieu thc ban a vao, cac ket
qua tnh toan, va cac ch bao a dang.

Neu mot ch bao xuat hien ben phai cua ket qua tnh toan,
ieu o ngha la ket qua tnh toan c hien th con tiep tuc sang
va
e cuon hien th ket qua tnh toan.
ben phai. Hay dung

Neu ch bao xuat hien ben phai cua bieu thc a vao, ieu o
ngha la tnh toan c hien th con tiep tuc sang ben phai. Hay
va
e cuon hien th bieu thc a vao. Lu y rang
dung
neu ban muon cuon bieu thc a vao trong khi ca ch bao va
trc het va roi dung
cung c hien th, ban se can nhan
e cuon.
va

Cac ch bao hien th


Ch th

Ngha la:
Ban phm so a c dch chuyen bang viec nhan
. Ban phm so se khong dch chuyen va ch
phm
bao nay se bien mat khi ban nhan mot phm.
6

Phng thc a vao kieu ch a c chon bang


. Phng thc a vao kieu ch se
viec nhan phm
ton tai va ch bao nay se bien mat khi ban nhan mot
phm.
M

Co mot gia tr c lu gi trong bo nh oc lap.

STO

May tnh tay ang i a vao mot ten bien e gan


mot gia tr cho bien nay. Ch bao nay xuat hien sau khi
(STO).
ban nhan

RCL

May tnh tay ang i a vao mot ten bien e nh lai


gia tr cua bien o. Ch bao nay xuat hien sau khi ban
.
nhan

STAT

May tnh tay ang trong phng thc STAT.


n v goc mac nh la o.
n v goc mac nh la radian.
n v goc mac nh la grad.

FIX

Quy nh so ch so phan thap phan.

SCI

Quy nh so ch so hien th.

Math

Hien th t nhien c la lam dang thc hien th.

D lieu bo nh ve bieu thc cu c gi lai va co the


c dung lai, hoac co nhieu d lieu tren/di man
hnh hien thi.

Disp

Hien th ang hien nh ket qua trung gian cua tnh


toan a cau lenh.

ieu quan trong: Vi mot so kieu tnh toan can thi gian thc hien
lau, man hnh khong hien g het trong khi no ang thc hien cac tnh
toan ben trong.

Dung menu
Mot so thao tac cua may tnh tay c thc hien bang viec dung
menu. Nhan
hay
chang han, se hien th menu cac ham ap
dung c.
Sau ay la cac thao tac ban nen dung e chuyen oi gia cac menu:

Ban co the la mot khoan muc menu bang viec nhan phm so
tng ng vi so ben trai cua no tren man hnh menu.

Dau goc tren ben phai cua menu ngha la co menu khac pha
di menu hien thi. Dau ngha la co menu khac pha tren. Hay
e chuyen qua cac menu.
dung

e ong menu ma khong la cai g, nhan


7

Xac nh phng thc tnh toan


Khi ban muon thc hien kieu thao
tac nay:

Hay thc hien thao tac


phm:

Tnh toan chung

(COMP)

Tnh toan thong ke va hoi quy

(STAT)

Giai phng trnh

(EQN)

Sinh ra bang so da tren bieu thc

(TABLE)
(INEQ)

Giai bat phng trnh


Tnh t le

(RATIO)

Lu y: Phng thc tnh toan mac nh la phng thc COMP.

Cai at cho may


Trc het thc hien thao tac phm sau e hien th menu thiet lap:
(SETUP). Tiep o, dung
va
va phm so e lap cau hnh
cai at ban muon.
Cai at co gach di (___) la mac nh khi au.
MthlO

LinelO

Xac nh dang thc hien th.

Hien th T nhien (MthlO) lam cho phan


so, so vo t va cac bieu thc khac c hien
th nh chung c viet tren giay.
Hien th Tuyen tnh (LinelO) lam cho phan
so, so vo t va cac bieu thc khac c hien
th tren mot hang.
Lu y: May tnh tay t ong chuyen sang Hien th tuyen tnh bat k
khi nao ban vao phng thc STAT. Trong tai lieu nay, k hieu
ben canh thao tac mau ch ra Hien th t nhien, trong khi k hieu
ch ra Hien th tuyen tnh.
Deg
Rad
Gra
Xac nh o, radian hay grad la n
v goc cho viec a vao gia tr va hien th ket qua tnh toan.
ng canh thao tac mau ch
Lu y: Trong tai lieu nay, k hieu
ra o, trong khi k hieu
ch ra radian.
Fix
Sci
qua tnh toan.

Norm

Xac nh so ch so e hien th ket

Fix: Ban nhap so trong khoang t 0 en 9 e an nh ch so phan


thap phan trong ket qua tnh toan c hien th. Ket qua tnh toan
8

c lam tron ti ch so a xac nh trc khi c hien th.


V du:

100 7 = 14,286 (Fix 3)


14,29

(Fix 2)

Sci: Ban nhap so trong khoang t 0 ti 9 e an nh ch so hien th.


Ket qua tnh toan c lam tron ti ch so a xac nh trc khi
c hien th.
V du:

1 7 = 1,4286 101
1,429 101

(Sci 5)
(Sci 4)

Norm: La chon mot trong hai cach thiet at san co (Norm 1, Norm
2) xac nh ra mien ma ket qua se c hien th theo dang thc
khong luy tha. Ben ngoai mien a xac nh, cac ket qua c hien
th bang viec dung dang thc luy tha.
> 1010 Norm 2: 109 > |x|, |x| =
> 1010
Norm 1: 102 > |x|, |x| =

V du:

1 200 = 5 103

(Norm 1)

0,005

(Norm 2)

ab/c
d/c Xac nh hoac hon so (ab/c) hoac
phan so (d/c) dung e hien th phan so trong ket qua tnh toan.
CMPLX
a + bi ;
r
Xac nh hoac toa o e
cac (a+bi) hoac toa o cc (r ) cho cac nghiem giai bang EQN.
STAT
ON ;
OFF
Xac nh co hay khong e
hien th cot FREQ khi bien soan trong STAT.
Rdec
ON ;
OFF
Xac nh lieu co hien th ket
qua tnh toan dung dang thc thap phan tuan hoan hay khong.
Disp
Dot ;
Comma Xac nh lieu co hien th
dau cham hay dau phay vao v tr dau cham thap phan. Dau cham
bao gi cung c hien th trong khi a vao.
Lu y: Khi dau cham c la lam dau cham thap phan, dau phan
cach cho phan thap phan cua ket qua la dau cham(.). Khi dau phay
c la, dau ngan cach la dau phay (,).
CONT
ieu chnh tng phan hien th. Xem ieu
chnh o tng phan hien th e biet chi tiet.

Khi au thiet b may tnh tay


Thc hien thu tuc sau e khi ong lai may tnh, se chuyen phng
thc tnh toan sang COMP va chuyen tat ca cac thiet at khac, ke ca
thiet at menu thiet at, ve mac nh khi au cua chung.
9

(CLR)

(Setup)

(Yes)

a vao bieu thc va gia tr


Quy tac a vao c ban
Cac tnh toan co the a vao theo cung dang nh chung c viet.
trnh t u tien cua viec a vao tnh toan se c
Khi ban nhan
t ong tnh va ket qua se xuat hien tren hien th.
4

4 sin30 (30 + 10 3) = 120


3
30
30
10
3

1
2
1

Phai a vao dau ngoac tron ong cho sin, sinh va cac ham
khac co cha dau ngoac tron.
Nhng k hieu nhan () co the c bo i. K hieu nhan co
the c bo i khi no xuat hien ngay trc mot dau ngoac
tron m, ngay trrc sin hay ham khac co cha dau ngoac
tron, ngay trc ham Ran# (so ngau nhien), hay ngay trc
bien (A, B, C, D, E, F, M, X, Y), hay e.
Dau ngoac trong ong ngay trc phep toan
co the c
bo i
V du a vao va bo cac phep toan
du tren
30
30
10
3
4

va

trong v

Lu y: Neu tnh toan tr nen dai hn chieu rong man hnh trong khi
a vao, man hnh se t ong cuon sang ben phai va ch bao se
xuat hien tren hien th. Khi ieu nay xay ra, ban co the cuon lai sang
va
e di chuyen con tro. Khi Hien
ben trai bang viec dung
th tuyen tnh c la, nhan
se lam con tro nhay len cho bat au
se lam cho con tro nhay ve cuoi. Khi
cua tnh toan, con nhan
trong khi con tro cuoi cua tnh
Hien th t nhien c la, nhan
toan a vao se lam cho no nhay ve cho bat au, trong khi nhan
khi con tro cho bat au se lam cho no nhay ve cuoi. Ban co the
a vao ti 99 byte. Mot so ham oi hoi ti 13 byte. Con tro se thay
oi hnh dang sang khi co 10 byte hay t hn cua phan con lai c
phep a vao. Neu ieu nay xay ra, hay ket thuc viec a vao tnh
.
toan roi nhan
10

Trnh t u tien tnh toan


Trnh t u tien cua tnh toan a vao c tnh theo quy tac di ay.
Khi u tien cua hai bieu thc la nh nhau, tnh toan c thc hien t
trai sang phai.
Th nhat
Th hai
Th ba
Th t

Bieu thc trong dau ngoac tron


Cac ham yeu cau oi ben phai va dau ngoac tron
ong ) theo sau oi so.
Cac ham co i theo sau gia tr a vao (x2, x3, x1, x!,
, , , , %), luy tha (x ), can ( )

o o r g

Phan so
Dau am ()

Th nam

Lu y: Khi bnh phng mot gia tr am (nh 2), gia tr


c bnh phng phai c bao trong ngoac tron (
2
). V x2 co u tien cao hn dau am,
2
se gay ra viec bnh phng
viec a vao
2 va do o gan them dau am vao ket qua. Bao gi cung
hay lu tam ti trnh t u tien, va bao cac gia tr am
trong ngoac tron khi c yeu cau.

Th sau

Cac gia tr c c lng theo phng thc STAT ( x ,


y , x 1, x 2)

Th bay

Phep nhan cho dau phep nhan b bo i

Th tam

Phep chnh hp (nPr), phep to hp (nCr)

Th chn

Phep nhan, phep chia (, )

Th mi Phep cong, phep tr (+, )

a vao bang hien th t nhien


La chon Hien th t nhien lam cho co kha nang a vao va hien th cac
phan so va nhng ham nao o (log, x2, x3, x ,

10 , e , Abs) nh chung c viet trong sach giao khoa cua ban.

11

x1 ,

2+ 2
1+ 2
2

ieu quan trong: Mot so kieu bieu thc co the lam cho chieu cao
cua cong thc tnh toan ln hn dong hien th. Chieu cao cho phep toi
a cua mot cong thc tnh toan la hai man hnh hien th (31 cham
2). a vao them na se tr thanh khong the c neu chieu cao cua
tnh toan ban a vao vt qua gii han c phep. Viec long cac
ham va cac dau ngoac la c phep. Viec a vao them na se tr
thanh khong the c neu ban long qua nhieu ham va/hoac cac dau
ngoac. Neu ieu nay xay ra, hay chia tnh toan thanh nhieu phan va
tnh tng phan mot cach tach biet.
Lu y: Khi ban nhan
va thu c ket qua tnh toan bang viec
dung hien th t nhien th mot phan cua bieu thc ban a vao co the
b cat mat. Neu ban can xem lai toan bo bieu thc a vao, nhan
va roi dung
va
e cuon bieu thc a vao.

Dung gia tr va bieu thc lam oi


(ch Hien th t nhien)
Gia tr hay bieu thc ban a vao co the c dung nh oi cua mot
7
ham. Sau khi ban a a vao chang han
, ban co the lam no thanh
6
oi cua

, tao thanh

a vao 1 +

7
.
6
7
6

7
va roi oi no thanh 1 +
6

6
(INS)

Nh ch ra tren, gia tr hay bieu thc ben phai cua con tro sau khi
(INS) c nhan tr thanh oi cua ham c xac nh tiep
o. Mien c bao quanh nh la tat ca cho ti dau m ngoac au tien
12

ben phai, neu nh co, hay moi th cho ti ham au tien ben phai
(sin(30), log2(4), v.v.).
Kha nang nay co the c dung cung vi cac ham sau:
( ),
( 10 ),
( e ),
,
,
(

,
3

),

log

Phng thc a vao ghi e (ch Hien th tuyen tnh)


Ban co the la hoac phng thc a vao chen them hoac ghi e,
nhng ch khi che o Hien th tuyen tnh c la. Trong phng thc
ghi e, van ban ban a vao thay the cho van ban v tr con tro. Ban
co the chuyen qua lai gia cac phng thc chen them va ghi e
bang viec thc hien thao tac:
(INS). Con tro xuat hien nh
trong phng thc chen them va nh trong phng thc ghi e.
Lu y: Hien th t nhien bao gi cung dung phng thc chen them,
cho nen thay oi dang thc hien th t Hien th tuyen tnh sang Hien
th t nhien se t ong chuyen vao phng thc chen them.

Sa cha va xoa bieu thc


Xoa mot k t hay ham: Chuyen con tro e no nam trc tiep ngay
. Trong
ben phai cua k t hay ham ban muon xoa, va roi nhan
phng thc ghi e, chuyen con tro e cho no nam trc tiep di k t
hay ham ban muon xoa, va roi nhan
.
e chen mot k t hay ham vao tnh toan: Dung
va
e
chuyen con tro ti v tr ban muon chen k t hay ham va roi a no
vao. Bao gi cung hay chac chan dung phng thc chen neu Hien
th tuyen tnh c la.
Xoa tat ca tnh toan ban a vao: Nhan

Tnh toan thap phan tuan hoan


May tnh cua ban dung so thap phan tuan hoan khi ban a vao gia
tr. Ket qua tnh toan cung co the c hien th bang viec dung dang
thc thap phan tuan hoan bat k khi nao ap dung c.

a vao so thap phan tuan hoan


Khi a vao so thap phan tuan hoan, nhan
(()) trc khi
a vao dau cham cua no va roi a vao dau cham cho gia tr ket
thuc. e a vao so thap phan tuan hoan 0.909090...(0.(90)), thc
hien thao tac sau: 0
(()) 90.

13

ieu quan trong: Neu gia tr bat au bang phan nguyen (nh:
12,3123123...), ng a phan nguyen vao khi a vao chu k (12,(312)).
a vao so thap phan tuan hoan la co the ch khi Hien th t nhien
c la.

e a vao 0.33333...(0.(3))
0

(())

3
e a vao 1.428571428571... (1.(428571))
1

(())

428571
e tnh 1,(031) + 2,(312)
1
2

(()) 021
(()) 312

Ket qua tnh toan c hien th nh


gia tr thap phan tuan hoan:
Lu y: Ban co the xac nh ti 14 v tr thap phan cho chu k thap
phan tuan hoan. Neu ban a vao nhieu hn 14 v tr thap phan, gia
tr nay se b x l nh so thap phan ket thuc va khong phai la phan so
thap phan tuan hoan. a vao gia tr thap phan tuan hoan co the
c thc hien bat ke thiet at Rdec tren menu thiet at.

Hien th ket qua tnh toan nh gia tr thap phan tuan hoan
Ket qua tnh toan co the c hien th nh gia tr thap phan tuan hoan
se c hien th nh vay khi ON c la cho thiet at Rdec tren
se quay vong gia cac dang thc ket
menu thiet at. Nhan phm
14

qua tnh toan nh c neu di ay.


Phan so

So thap phan tuan hoan

Gia tr thap phan tng ng cho thiet at hien th


(Norm, Fix, Sci)

hay
Gia tr thap phan tng ng cho thiet at hien th
(Norm, Fix, Sci)
So thap phan tuan hoan

Phan so

1
= 0,(142857) = 0,1428571429 (Norm 1)
7

Hien th nh so thap phan tuan hoan:


Gia tr thap phan tng ng vi thiet at Norm 1:
Tr lai dang thc hien th ban au (phan so):

17=

1
= 0,(142857) = 1,1428571429 (Norm 1)
7

Hien th nh phan so:


Hien th nh so thap phan tuan hoan:
Tr lai dang thc hien th ban au (Norm 1):

15

1
= 0,(142857) = 0,1428571429 (Norm 1)
7

Hien th nh so thap phan tuan hoan:


Gia tr thap phan tng ng vi thiet at Norm 1:
Cho lai dang thc hien th ban au (phan so):
1 7 = 0,1428571429 (Norm 1) = 0,(142857) =
1

1
7

Hien th nh phan so:


Hien th nh so thap phan tuan hoan:
Tr lai dang thc hien th ban au (Norm 1)

ieu kien hien th ket qua tnh toan nh so thap phan


tuan hoan
Neu ket qua tnh toan thoa man cac ieu kien sau , nhan
th no nh gia tr thap phan tuan hoan.

se hien

Tong so ch so c dung trong phan so co hon so (ke ca so


nguyen, t so, mau so, va k hieu phan tach) phai khong qua 10.

Kch c d lieu cua gia tr c hien th nh so thap phan tuan


hoan phai khong ln hn 99 bytes. Tng gia tr va dau cham thap
phan yeu cau mot byte, va tng ch so cua phan tuan hoan yeu
cau mot byte. Chang han so sau ay se yeu cau toan bo 8 byte (4
byte cho gia tr, 1 byte cho dau cham thap phan, 3 byte cho phan
tuan hoan): 0,(123)

Lu y: e biet thong tin ve viec chuyen dang thc hien th cua ket
qua tnh toan khi OFF c la cho thiet at Rdec tren menu thiet at,
xem Chuyen ket qua tnh toan.
16

V du ve so thap phan tuan hoan


0,(3) + 0,(45) = 0,(78)
0
0

(()) 3
(()) 45

1,(6) + 2,(8) = 4,(5)


1
2

(()) 6
(()) 8

e xac nhan ieu sau: 0,(123) =


0,(12345) =

12345
99999

123

1234

12345

123
1234
, 0,(1234) =
,
999
9999

999

9999

99999

Chuyen ket qua tnh toan


Khi Hien th t nhien c la, moi lan nhan
se chuyen ket qua tnh
toan c hien th hien tai sang dang thc phan so va dang thc thap phan
cua no, dang thc can
va dang thc thap phan cua no, hay dang thc
17

va dang thc thap phan cua no.


1
= 0,5235987756
6

6=

()
( 2 + 2)
2

6
3 =

0.5235987756

6 + 2 3 = 5,913591358

6 +2 3

5.913591358

Khi Hien th tuyen tnh c la, tng viec nhan


se chuyen ket
qua tnh toan hien thi sang cac dang thap phan va dang phan so cua
no.
1 5 = 0,2 =
1
1
1

1
5

0.2

1 5

1 5

0.2

4
1
=
= 0,2
5
5

ieu quan trong: Tuy theo kieu tnh toan ang tren man hien th khi
, qua trnh chuyen oi co the mat mot chut thi
ban nhan phm
gian e thc hien. Vi mot so ket qua tnh toan, nhan phm
se
khong chuyen oi gia tr a hien th. Khi ON c la cho Rdec tren
menu thiet lap, nhan
se chuyen ket qua tnh toan sang dang thap
phan tuan hoan. e biet chi tiet, xem Tnh toan thap phan tuan
hoan.
Lu y: Vi Hien th t nhien, nhan
thay v
sau khi a vao
mot tnh toan se hien th ket qua tnh toan di dang thap phan. Nhan
sau o se chuyen ket qua tnh toan sang dang thap phan tuan
hoan, dang phan so hay dang . Dang
xuat hien trong trng hp nay.

cua ket qua se khong

Tnh toan c ban


Tnh toan phan so
Lu y rang phng phap a vao la khac nhau, tuy theo lieu ban
ang dung Hien th t nhien hay Hien th tuyen tnh.

18

2 1 7

3 2 6

2
2

hay

1 1
=
2 2

7
6

7
6

7 6

)3

1
2

1 2

3
2

43

(
4

Lu y: Phan so co hon so va gia tr thap phan trong mot tnh toan


khi Hien th tuyen tnh c la se gay ra ket qua c hien th nh
gia tr thap phan. Phan so trong ket qua tnh toan c hien th sau
khi c rut gon ve dang toi gian cua chung.
e chuyen ket qua tnh toan gia phan so khong toi gian va dang
thc phan so hon so: thc hien thao tac chu chot sau
b
d
(a )
c
c
e chuyen ket qua tnh toan gia phan so va dang thc thap
phan: Nhan

Tnh phan tram


a vao mot gia tr va nhan
thanh so phan tram.
150 20% = 30

(%) lam cho gia tr a vao tr


150

(%)

30

880

(%)

75

15

(%)

2875

25

(%)

2625

20

Tnh phan tram nao cua 880 la 660. (75%)


660
Tang 2500 len 15%. (2875)
2500

2500

Giam 3500 i 25%. (2625)


3500

3500

Tnh toan o, phut, giay (o, phut, giay)


Thc hien phep cong hay tr gia cac gia tr o, phut, giay, hay phep
nhan va chia gia cac gia tr o, phut, giay va gia tr thap phan se lam
cho ket qua c hien th theo gia tr o, phut, giay. Ban cung co the
chuyen oi gia o, phut, giay va thap phan. Sau ay la dang thc
phut
giay
.
a vao cho gia tr o, phut, giay: o
19

Lu y: Ban bao gi cung phai nhap gia tr cho o va phut, cho du gia
tr o la khong.
22030+ 3930 = 30000
2

20

30

39

300

30

Chuyen 21518 sang dang tng ng thap phan.


2

15

21518

18

(Chuyen oi he sau mi sang thap phan.)

2.255

(Chuyen oi he thap phan sang he sau mi.)

21518

Ket noi nhieu bieu thc


Ban co the dung k t hai cham (:) e noi hai hay nhieu bieu thc va
.
thc hien chung t trai sang phai khi ban nhan
3+3:33

(:) 3

6
9

Dung k phap k thuat


Mot thao tac phm n gian bien oi gia tr c hien th sang k phap
k nghe.
Bien oi gia tr 1234 sang k phap k nghe, dch chuyen dau
cham thap phan sang phai
1234
1234
1.234103
123410
Bien oi gia tr 123 sang k phap k nghe, dch chuyen dau cham
thap phan sang trai.
123
123
()

0.123103

()

0.000123106

Hien lai bieu thc cu


Trong Phng thc COMP, may tnh tay nh xap x 200 byte d lieu
cho tnh toan mi nhat. Ban co the cuon qua noi dung bieu thc cu
va
.
bang viec dung
1+1=2

2+2=4

3+3=6

(Cuon ngc lai.)

(Cuon ngc lai na.)

20

Lu y: D lieu c tnh toan trc o c xoa khi nhan


, khi ban
thay oi phng thc tnh toan khac, khi ban thay oi dang thc hien
th, hay bat k khi nao ban thc hien thao tac at tai reset.

Chay lai
Khi ket qua tnh toan ang tren man hien th, ban co the nhan
e sa oi bieu thc ban a dung cho tnh toan trc.
4

43 + 2,5 = 14,5
43 7,1 = 4,9 (Tiep tuc)

va

2.5

14.5

7.1

4.9

Lu y: Neu ban muon sa mot tnh toan khi ch bao ang ben
phai cua hien th ket qua tnh toan(xem oc hien th), nhan
va
va
e cuon tnh toan.
roi dung

Bo nh ket qua (Ans)


Ket qua tnh toan cuoi cung thu c la c lu trong bo nh Ans
(tra li). Noi dung bo nh Ans c cap nhat khi ket qua tnh toan mi
c hien th.
e chia ket qua cua 3 4 cho 30
3
(Tiep tuc)
123 + 456 = 579

123

4
30
456

789 579 = 210


(Tiep tuc) 789,

Cac bien (A, B, C, D, E, F, X, Y)


May tnh tay cua ban co tam bien at san co ten A, B, C, D, E, F, X va
Y. Ban co the gan gia tr cho cac bien va cung co the dung cac bien
nay trong tnh toan.
e gan ket qua cua 3 + 5 cho bien A
3

(STO)

(A)

e nhan noi dung cua bien A vi 10


(A)

(Tiep tuc)
e nh lai noi dung cua bien A (Tiep tuc)
e xoa noi dung cua bien A
21

(STO)

80

10
(A)

(A)

Bo nh oc lap (M)
Ban co the cong ket qua tnh toan hay tr ket qua t bo nh oc lap.
Ch M xuat hien tren man hien th khi co gia tr khac khong c lu
trong bo nh oc lap.
e xoa noi dung cua M

(STO)

e cong ket qua cua 10 5 vao M

(Tiep tuc)

10

(M)

0
50

e tr ket qua cua 10 + 5 t M


(Tiep tuc)
e nh noi dung cua M

10

(Tiep tuc)

(M)

15

(M)

35

Lu y: Bien M c dung cho bo nh oc lap.

Xoa noi dung cua moi bo nh


Bo nh Ans, bo nh oc lap, va noi dung bien c duy tr cho du ban
nhan
, thay oi phng thc tnh toan, hay tat may tnh. Hay thc
hien thu tuc sau khi ban muon xoa noi dung cua tat ca cac bo nh.
(CLR)

(Memory)

(Yes)

Tnh ham
Vi cac thao tac thc tai dung tng ham, xem muc V du theo sau
danh sach di ay.
: c hien th la 3,141592654, nhng = 3,14159265358980
c dung cho tnh toan noi bo.
e: e c hien th la 2,718281828, nhng e = 2,71828182845904
c dung cho tnh toan noi bo.
sin, cos, tan, sin1, cos1, tan1: Cac ham lng giac. Xac nh n v
1.

goc trc khi thc hien tnh toan . Xem


1

sinh, cosh, tanh, sinh , cosh , tanh : Cac ham hyperbolic. a


. Thiet at n v goc
vao ham t menu xuat hien khi ban nhan
khong anh hng ti tnh toan. Xem

2.

o r g

, , : Cac ham nay xac nh n v goc . o xac nh o, r radian, va g


grad. a vao mot ham t menu xuat hien khi ban thc hien thao tac
(DRG ). Xem

phm sau:

10 ,

3.

: Ham luy tha. Lu y rang phng phap a vao la khac


nhau tuy theo lieu ban ang dung Hien th t nhien hay Hien th tuyen
tnh. Xem

4.

log: Ham lo ga rit. Dung phm


e a vao logab nh log(a, b). C
so 10 la thiet at mac nh neu ban khong a vao cai g cho a. Phm
22

cung co the c dung cho a vao, nhng ch khi Hien th t


nhien c la. Trong trng hp nay, ban phai a vao mot gia tr
log

5.

cho c so. Xem

In: Lo ga rit t nhien c so e. Xem


x2, x3, x ,

6.

, x1 : Luy tha, can, va lay nghch ao. Lu y

rang phng phap a vao cho x ,

va

la khac nhau tuy


7.

theo lieu ban dung Hien th t nhien hay Hien th tuyen tnh. Xem

Lu y: Cac ham sau khong the c a vao theo trnh t ke tiep: x2,
chang han,
cuoi cung se
x3, x, x1. Neu ban a vao 2
2

b bo qua. e a vao 22 , a vao 2


.
nhan

, nhan phm

va roi

Pol, Rec: Pol chuyen oi toa o ch nhat sang toa o cc, trong khi
8.
Rec chuyen oi toa o cc sang toa o ch nhat. Xem
Xac nh n v goc trc khi thc
hien tnh toan.
Ket qua tnh toan cho r va va cho
x va y tng phan t c gan tng
ng cho cac bien X va Y. Ket qua
tnh toan c hien th trong
o
pham vi 180o <
180
x!: Ham giai tha. Xem

9.

Abs: Ham gia tr tuyet oi. Lu y rang phng phap a vao la khac
nhau tuy theo lieu ban dung Hien th t nhien hay Hien th tuyen tnh.
Xem

10.

Ran#: Sinh ra so gia ngau nhien 3 ch so be hn 1. Ket qua c


hien th nh phan so khi Hien th t nhien c la. Xem

11.

Ranlnt#: Nhap a, b vao ham Ranlnt#(a, b), ham sinh ra so nguyen


ngau nhien ben trong pham vi a ti b. Xem

12.

nPr, nCr: Ham chnh hp (nPr) va ham to hp (nCr). Xem

13.

Rnd: oi cua ham nay c tao nen t gia tr thap phan va roi c
lam tron tng ng vi so hien thi cua thiet at cac ch so hien th
(Norm, Fix hay Sci).Vi Norm1 hay Norm2, oi c lam tron ti 10
ch so. Vi Fix va Sci, oi c lam tron ti ch so a xac nh. Khi
Fix 3 la thiet at ch so hien th chang han, ket qua cua 10 3 c
hien th la 3,333, trong khi may tnh tay van gi gia tr
3,33333333333333 (15 ch so) ben trong cho tnh toan. Trong trng
23

hp cua Rnd (10 3) = 3,333 (vi Fix 3), ca hai gia tr c hien th
va gia tr ben trong cua may tnh tay eu tr thanh 3,333. Bi v ieu
nay mot chuoi cac tnh toan se tao ra cac ket qua khac nhau tuy theo
lieu Rnd c dung (Rnd(10 3) 3 = 9,999) hay khong c dung
14.

((10 3 3 = 10,000). Xem

Lu y: Dung cac ham co the lam cham viec tnh toan, ieu o co the
lam tre hien th ket qua. ng thc hien thao tac ke tiep nao trong khi
ch i ket qua tnh toan xuat hien. e ngat tnh toan ang dien ra
.
trc khi ket qua xuat hien, nhan

V du
30

0.5

(sin-1)0.5

30

o
1 sin 30 = 0,5

sin-10,5 = 30o
2 sinh 1 = 1,175201194
cosh11 = 0

(sinh) 1

1.175201194

(cosh1) 1

o
o
3 /2 radians = 90 , 50 grads = 45

()

(DRG )

(DRG )

50

(r)

90

45

()

4 e tnh toan e 2 ti ba ch so co ngha (Sci 3)


(SETUP)
(Sci)
(e) 5

2.97102

(e) 5

2.97102

5 log101000 = log 1000 = 3


log216 = 4
2

1000

(,) 16

log

16

e tnh ln 90 (= loge90) ti ba ch so co ngha (Sci 3)

(SETUP)

(Sci)

3
7 1,2 10 = 1200
(1+1)2+2 =16

1.2
1

(5 ) = 15625

1200

16

15625

( ) 5

32 = 2

32

( ) 32

5
e tnh

10

2 3

4.50100

90

2 3 = 3 2 = 4,242640687...) ti ba v tr thap phan (Fix 3)

(SETUP)

(Fix)

2
24

4.243
2

4.243

8 e chuyen oi toa o ch nhat ( 2 , 2 ) sang toa o cc


(Pol)
2
(,)
2
r= 2, =45
(Pol)

(,)

r=2
= 45

e chuyen oi toa o cc ( 2 , 45o) sang toa o ch nhat


(Rec)

9 (5 + 3) ! = 40320

(,) 45
3

X= 1, Y=1
(x!)

40320

10 |2 7| 2 = 10
2
2

10

10

11 e thu c so nguyen ba ch so ngau nhien


1000
(Ran#)

459
48
117

(Ket qua c neu ay ch vi muc ch minh hoa. Ket qua thc se khac.)

12 e sinh ra so nguyen ngau nhien trong pham vi 1 ti 6


(Ranlnt)1
(,)6

2
6
1

(Ket qua c neu ay ch vi muc ch minh hoa. Ket qua thc se khac.)

13 e xac nh so cac chnh hp va to hp co the khi la bon


ngi t mot nhom 10 ngi
Chnh hp: 10

(nPr) 4

5040

To hp: 10

(nCr) 4

210

14 e thc hien cac tnh toan sau ay khi Fix 3 c la cho so


cac ch so hien th: 10 3 3 va Rnd (10 3) 3
(SETUP)
(Fix)
3
3
10
10.000
0 (Rnd) 10
3
3
9.999

Tnh toan thong ke (STAT)


e bat au tnh toan thong ke, hay thc hien thao tac phm
(STAT) e a vao phng thc STAT va roi dung man hnh xuat
hien e la kieu tnh toan ban muon thc hien
25

e la kieu tnh toan thong ke:


Nhan phm nay:
(Cong thc hoi quy c neu trong ngoac)
Bien n (X)

(1-VAR)

Bien oi (X, Y), hoi qui tuyen tnh

(y = A + Bx)
2

(A + BX)

Bien oi (X, Y), hoi quy bac hai (y = A + Bx + Cx )

(_ + CX2)

Bien oi (X, Y), hoi quy logarit

(y = A + Blnx)

(ln X)

Bien oi (X, Y), hoi quy ham mu e

(y = AeBx)

(e^X)

Bien oi (X, Y), hoi quy ham mu ab

(y = ABx)
B

Bien oi (X, Y), hoi quy ham luy tha


Bien oi (X, Y), hoi quy nghch ao
Nhan bat k phm tren (
ke Stat.

ti

(y = Ax )
(y = A + B/x)

(AB^X)
(AX^B)
(1/X)

) cho hien th Bo soan thao thong

Lu y: Khi ban muon thay oi kieu tnh toan sau khi vao phng thc
STAT, hay thc hien thao tac phm
(STAT)
(Type) e
hien th man hnh la kieu tnh toan.

a d lieu vao
Dung bo soan thao thong ke Stat e a d lieu vao. Thc hien thao
tac phm sau e hien th bo soan thao thong ke Stat:
(STAT)
(Data).
Bo soan thao thong ke Stat cung cap 80 dong cho vao d lieu khi co
mot cot X, 40 dong khi co cot X va FREQ hay cot X va Y, hay 26
dong khi co cot X, Y va FREQ.
Lu y: Dung cot FREQ (tan xuat) e a vao so lng (tan xuat) cua
cac khoan muc d lieu ong nhat. Hien th cot FREQ co the c bat
len (c hien th) hay tat i (khong c hien th) bang viec dung
thiet at dang thc Stat tren menu thiet at.
1

e la hoi quy tuyen tnh va a vao d lieu sau:


(170, 66), (173, 68), (179, 75)
(STAT)

170

173

(A + BX)

179

66

68
26

75

ieu quan trong: Tat ca d lieu hien a vao trong bo soan thao
thong ke Stat eu b xoa i bat k khi nao ban ra khoi STAT, chuyen
gia kieu tnh toan thong ke bien n va bien oi, hay thay oi thiet
at dang thc Stat tren menu thiet at. Thao tac sau khong c ho
,
(M),
(STO).
tr bi bo soan thao thong ke Stat:
Pol, Rec, va a cau lenh cung khong the c a vao vi bo soan
thao thong ke Stat.
e thay oi d lieu trong mot o: Trong bo soan thao thong ke Stat,
chuyen con tro ti o co cha d lieu ban muon thay oi, a vao d
.
lieu mi, va the roi nhan
e xoa mot dong: Trong bo soan thao thong ke Stat, chuyen con tro
.
ti dong ban muon xoa va roi nhan
e chen them mot dong: Trong bo soan thao thong ke Stat, chuyen
con tro ti v tr ban muon chen dong va roi thc hien thao tac phm
(STAT)
(Edit)
(lns).
sau:
e xoa tat ca noi dung cua bo soan thao Stat: Trong bo soan thao
thong ke Stat, thc hien thao tac phm sau:
(STAT)
(Edit)
(Del-A).

Goi ket qua thong ke


e goi ket qua thong ke, nhan
khi trong bo soan thao thong ke
Stat va the roi nh bien thong ke (xn, x2 v.v) ban muon. Cac bien
thong ke c ho tr va cac phm ban phai nhan e nh chung c
neu di ay. Vi cac tnh toan thong ke bien n, cac bien c
anh dau bang dau sao () la co san.
Sum: x2, x, y2, y, xy, x3, x2y, x4
(STAT)

(Sum)

ti

Tong tan so: n, Trung bnh: x * , y , o lech chuan khong gian


mau: x n, y, o lech chuan mau: xn1, yn1
(STAT)

(Var)

ti

Gia tr toi thieu: minX, minY, Gia tr toi a: maxX, maxY


(STAT)

(MinMax)

ti

(Khi tnh toan thong ke bien n c la)


(STAT)

(MinMax)

ti

(Khi tnh toan thong ke bien oi c la)


He so hoi quy: A, B, He so tng quan: r, Gia tr c lng: x , y
(STAT)

(Reg)

ti
27

He so hoi quy cho Hoi quy bac hai: A, B, C, gia tr c lng: x 1, x 2, y


(STAT)

(Reg)

ti

Xem bang cho bat au cua muc nay cua tai lieu nay ve cac cong
thc hoi quy.
x , x 1, x 2 va y khong phai la bien. Chung la cac ch lenh co kieu
nhan mot oi ngay trc chung. Xem Tnh gia tr c lng e
biet them thong tin.
2 e a vao d lieu bien n x = 1, 2, 2, 3, 3, 4, 4, 5, dung
cot FREQ e xac nh so lap cho tng khoan muc xn; freqn
= 1;1, 2;2, 3;3, 4;2, 5;1, va tnh gia tr trung bnh va o lech
chuan khong gian mau.
(SETUP)
(STAT)
(ON)
(STAT)
(1-VAR)
2
3
4
5
1
1
2
3
2
(STAT)
(STAT)

(Var)
(Var)

(x)
(xn)

Ket qua: Trung bnh: 3. o lech chuan khong gian mau: 1,154700538
3 e tnh toan cac he so tng quan hoi quy tuyen tnh va hoi
quy logarit cho d lieu bien oi sau va xac nh cong thc hoi
qui cho tng quan manh nhat: (x, y) = (20, 3150), (110,
7310), (200, 8800), (290, 9310). Xac nh Fix 3 (ba v tr thap
phan) cho ket qua.
(SETUP)
(STAT)
(OFF)
(SETUP)
(Fix)
(STAT)
(A+ BX)
110
200
290
20
3150
7310
8800
9310
(STAT)

(Reg)

(STAT)

(Type)

(STAT)

(Reg)

(r)
(ln X)
(r)

(STAT)

(Reg)

(A)

(STAT)

(Reg)

(B)

Ket qua: He so tng quan hoi quy tuyen tnh: 0,923


He so tng quan hoi quy logarit: 0,998
28

Cong thc hoi quy logarit: y = 3857,984 + 2357,532lnx

Tnh gia tr c lng


Da tren cong thc hoi quy thu c bang tnh toan thong ke bien oi,
gia tr c lng cua y co the c tnh toan theo gia tr x a cho. Gia
tr x tng ng (hai gia tr, x1 va x2, trong trng hp hoi quy bac hai)
cung co the c tnh toan cho gia tr cua y trong cong thc hoi quy.
e xac nh gia tr c lng cho y khi x = 160 trong cong
thc hoi quy c tao ra bi hoi quy logarit cua d lieu trong

3 , xac nh Fix 3 cho ket qua. (Thc hien thao tac sau
ay sau khi hoan thanh thao tac trong
(STAT)

160
Ket qua:

(Reg)

3).

( y )

8106.898

ieu quan trong: Tnh toan he so hoi quy, he so tng quan, va gia
tr c lng co the ton thi gian ang ke khi co so ln cac khoan muc
d lieu.

Tnh toan phng trnh (EQN)


Ban co the dung thu tuc sau trong phng thc EQN e giai he
phng trnh tuyen tnh ong thi vi hai hay ba an, phng trnh bac
hai, va phng trnh bac ba.
1. Nhan

(EQN) e vao phng thc EQN.

2. Tren menu xuat hien, la kieu phng trnh.


e la kieu tnh toan nay:

Nhan phm nay:

Phng trnh tuyen tnh ong thi vi


hai an

(anX + bnY = cn)

Phng trnh tuyen tnh ong thi vi


ba an

(anX + bnY + cnZ = dn)

Phng trnh bac hai

(aX2 + bX + c = 0)

Phng trnh bac ba

(aX3 + bX2 + cX + d = 0)

3. Dung bo soan thao he so Coefficient xuat hien e a vao cac gia


tr he so.

e giai 2x2 + x 3 = 0, chang han, nhan


1
cac he so (a = 2, b = 1, c= 3): 2

e thay oi gia tr he so ban a a vao, chuyen con tro ti o


thch hp, a vao gia tr mi, va roi nhan
.

Nhan

bc 2, va nhap
3
.

se xoa tat ca cac he so thanh khong.


29

ieu quan trong: Cac thao tac sau khong c ho tr bi bo soan


,
(M),
(STO). Pol, Rec
thao he so Coefficient:
va a cau lenh cung khong the c a vao bang bo soan thao
he so Coefficient.
.
4. Sau khi tat ca cac gia tr ban muon a c a vao, nhan

ieu nay se hien th nghiem. Moi lan nhan


se cho hien th
khi nghiem cuoi cung c hien th
mot nghiem khac. Nhan
se tr lai bo soan thao he so Coefficient.

Ban co the cuon gia cac nghiem bang viec dung cac phm
.
va

e tr ve bo soan thao he so Coefficient trong khi bat k nghiem


nao c hien th, nhan
.

Lu y: Cho du Hien th t nhien c la, nghiem cua phng trnh


tuyen tnh ong thi khong c hien th bang viec dung bat k dang
nao co cha
.
Cac gia tr khong the c chuyen oi thanh k hieu k thuat
tren man hnh nghiem.

Thay oi thiet at kieu phng trnh hien thi


Nhan
(EQN) va roi la mot kieu phng trnh t menu xuat
hien. Thay oi kieu phng trnh lam cho cac gia tr cua moi he so bo
soan thao he so Coefficient b oi thanh khong.

V du tnh toan phng thc EQN


x + 2y = 3, 2x + 3y = 4
(EQN)

(anX + bnY = cn)


1
2
3
2
3
4
(X=) 1
(Y=) 2

x y + z = 2, x + y z = 0, x + y + z = 4
(EQN)
1
1

(anX + bnY + cnZ = dn)


1
1
2
1
1
0
1
1
1
4
(X=) 1
(Y=) 2
(Z=) 3
30

x2 + x +

3
=0
4

(aX2 + bX + c = 0)

(EQN)
1

(X1=)

1
2
+
i
2
2

(X2=)

1
2

i
2
2

x2 2 2 x + 2 = 0
(EQN)
1

(aX2 + bX + c = 0)
2

(X=)

x3 2x2 x + 2 = 0
1

(EQN)
2

(aX3 + bX2 + cX + d = 0)
1
2

(X1=) 1
(X2=) 2
(X3=) 1

Tao ra bang so t ham (TABLE)


TABLE sinh ra bang so cho x va f(x) bang viec dung ham a vao f(x).
Thc hien cac bc sau e sinh ra bang so.
1. Nhan

(TABLE) e a vao phng thc TABLE.

2. a vao ham di dang f(x), bang viec dung bien X.

Hay chac a vao bien X (


(X)) khi sinh ra bang so. Bat
k bien nao khac hn X cung eu b x tr nh hang.

ieu sau ay khong the c dung trong ham nay: Pol, Rec.

3. ap lai li nhac xuat hien, hay a vao cac gia tr ban muon dung,
nhan
sau moi gia tr.
Vi li nhac:

a vao:

Start?

a vao gii han di cua X (mac nh = 1).


a vao gii han tren cua X (mac nh = 5).

End?

Lu y: Hay chac chan rang tr End luon luon ln


hn tr Start.
a vao bc tang ( mac nh = 1).

Step?

Lu y: Step xac nh cach gia tr Start phai tuan


t tang len khi bang so c sinh ra. Neu ban
xac nh Start = 1 va Step = 1, X se tuan t c
gan cac gia tr 1, 2, 3, 4 v.v e sinh ra bang so
cho ti khi gia tr End c at ti
31

a vao gia tr Step roi nhan


sinh ra va hien th bang so
tng ng vi cac tham bien ban xac nh.

Nhan
khi man hnh bang so c hien th se tr lai man hnh
a vao ham bc 2.
e sinh ra bang so cho ham f(x) = x2 +

1
< <
vi mien 1 = x = 1,
2

c tang theo bc 0,5


(TABLE)

(X)

0.5

Lu y: Ban co the dung man hnh bang so ch e xem cac gia tr.
Noi dung bang khong the c sa oi. Thao tac sinh bang so lam
cho noi dung cua bien X b thay oi.
ieu quan trong: Ham ban a vao cho viec sinh bang so b xoa i
bat k khi nao ban hien th menu thiet lap trong phng thc TABLE
va chuyen gia Hien th t nhien va Hien th tuyen tnh.

Tnh toan bat phng trnh (INEQ)


Ban co the dung thu tuc sau e giai bat phng trnh bac hai hay bat
phng trnh bac ba.
1. Nhan

(INEQ) e a vao phng thc INEQ.

2. Tren menu xuat hien, la kieu bat phng trnh.


e la kieu bat phng trnh nay:
Bat phng trnh bac hai
Bat phng trnh bac ba
3. Tren menu xuat hien, dung cac phm
hieu bat phng trnh va hng.

Hay nhan phm nay:


(aX2 + bX + c)
(aX3 + bX2 + cX + d)
ti

e la kieu k

4. Dung bo soan thao he so Coefficient xuat hien ra e a vao cac


gia tr he so.

e giai x2 + 2x 3 < 0 chang han, a vao cac he so a = 1,


2
3
.
b = 2, c = 3 bang viec nhan 1
32

e thay oi gia tr he so ban va a vao, chuyen con tro ti o


.
thch hp, a vao gia tr mi, va roi nhan

Nhan

se xoa tat ca cac he so ve khong.

Lu y: Cac thao tac sau khong c ho tr bi bo soan thao he so


,
(M),
(STO), Pol, Rec va a cau lenh
Coefficien:
cung khong c a vao bang bo soan thao he so Coefficien.
5. Sau khi tat ca cac gia tr a la nh ban mong muon, nhan

ieu nay se cho hien th nghiem.

e tr ve bo soan thao he so Coefficien trong khi nghiem ang


.
c hien th, nhan

Lu y: Cac gia tr khong the c chuyen oi sang k phap k nghe


tren man hnh nghiem.

Thay oi kieu bat phng trnh


Nhan
(INEQ) va the roi la chon kieu bat phng trnh t
menu xuat hien. Thay oi kieu bat phng trnh lam cho cac gia tr
cua moi he so trong bo soan thao he so Coefficien oi thanh khong.

V du tnh toan theo phng thc INEQ


x2 + 2x 3 < 0
(aX2 + bX + c)

(INEQ)

(aX2 + bX + c < 0)

>
x2 + 2x 3 = 0

(INEQ)

(aX2 + bX + c)
>

(aX2 + bX + c = 0)
2
3
1

33

Lu y: Nghiem c hien th nh c neu


ay khi Hien th tuyen tnh c la.
>
2x3 3x2 = 0

(aX3 + bX2 + cX + d)

(INEQ)

>

(aX3 + bX2 + cX + d = 0)
3
2

3x3 + 3x2 x > 0


(aX3 + bX2 + cX + d)

(INEQ)

(aX3 + bX2 + cX + d > 0)


3
3
1

Lu y: Cac nghiem c hien th nh neu


ay khi Hien th tuyen tnh c la.
Hien th nghiem ac biet

All xuat hien tren man hnh nghiem khi nghiem cua bat phng
trnh tat ca eu la so.
>0
x2 =

(aX2 + bX + c)

(INEQ)

>
(aX2 + bX + c = 0)
0
0
1

NOT FOUND xuat hien tren man hnh nghiem khi khong co
nghiem cho bat phng trnh (nh X2 < 0).

34

Tnh t so (RATIO)
Phng thc RATIO cho phep ban xac nh gia tr cua X trong bieu
thc t so a:b = X:d (hay a:b = c:X) khi cac gia tr a, b, c va d a c
biet. ieu sau ay ch ra thu tuc chung e dung RATIO.
1. Nhan

( RATIO) e a vao phng thc RATIO.

2. Tren menu xuat hien, la

(a:b = X:d) hay

(a:b = c:X).

3. Tren man hnh bo soan thao he so Coefficient xuat hien, a vao


cho ti 10 ch so cho tng gia tr c yeu cau (a, b, c, d).

e giai 3:8 = X:12, chang han, nhan


cac he so (a = 3, b = 8, d = 12): 3
8

bc 1, va roi nhap
12
.

e thay oi gia tr he so ban va a vao, chuyen con tro ti o


.
thch hp, a vao gia tr mi, va roi nhan

Nhan

se xoa tat ca cac he so thanh khong.

Lu y: Cac thao tac sau khong c ho tr bi bo soan thao he so


,
(M),
(STO). Pol, Rec va a cau
Coefficient:
lenh cung khong c a vao bang bo soan thao he so Coefficient.
4. Sau khi tat ca cac gia tr a la nh ban mong muon, nhan

ieu nay se cho hien th nghiem (gia tr cua X). Nhan


na se tr lai bo soan thao he so Coefficient.

.
lan

ieu quan trong: Loi Math ERROR se xuat hien neu ban thc hien
tnh toan trong khi 0 la d lieu nhap cho mot he so.

Thay oi kieu bieu thc t so


Vao lai phng thc RATIO va la kieu bieu thc t so ban muon co t
menu xuat hien. Thay oi kieu bieu thc t so lam cho cac gia tr cua
moi he so cua bo soan thao he so Coefficient oi thanh khong.

V du tnh toan phng thc RATIO


e tnh X trong t so 1 : 2 = X : 10
(RATIO)

(a:b=X:d) 1

35

10

e tnh X trong t so 1 : 2 = 10 : X
(RATIO)

(a:b=c:x) 1

10

Mien tnh toan, so ch so va o chnh xac


Mien tnh toan, so ch so c dung cho tnh toan ben trong, va o
chnh xac phu thuoc vao kieu tnh toan ban thc hien.

Mien tnh toan va o chnh xac


1 1099 ti 9,999999999 1099 hay 0

Mien tnh toan

So ch so cho tnh toan


15 ch so
ben trong
o chnh xac

Noi chung, 1 tai ch so th 10 cho tnh


toan n. o chnh xac cho hien th mu la
1 tai ch so t y ngha nhat. Sai so b tch
luy trong trng hp tnh toan lien tiep.

Mien a vao tnh toan ham va o chnh xac


Ham

sinx

cosx

tanx

Mien a vao
DEG

9
0<
= x < 9 10

RAD

0<
= x < 157079632,7

GRA

10
0<
= x 1 10

DEG

9
0<
= x < 9 10

RAD

0<
= x < 157079632,7

GRA

10
0<
= x 1 10

DEG

Nh sinx, ngoai tr khi |x| = (2n 1) 90.

RAD

Nh sinx, ngoai tr khi |x| = (2n 1) /2.

GRA

Nh sinx, ngoai tr khi |x| = (2n 1) 100.


36

sin1x
cos x
1

tan1x
sinhx
coshx
sinh1x
cosh1x

0<
= |x| <
=1
99
0<
= |x| <
= 9,999999999 10

0<
= |x| <
= 230,2585092
99
0<
= |x| <
= 4,999999999 10
99
1<
= |x| <
= 4,999999999 10

tanh1x

99
0<
= |x| <
= 9,999999999 10
1
0<
= |x| <
= 9,999999999 10

logx/lnx

99
0<x <
= 9,999999999 10

tanhx

10x
ex
x

9,999999999 1099 <


=x <
= 99,99999999
9,999999999 1099 <
=x <
= 230,2585092
100
0<
= x < 1 10

x2

|x| < 1 1050

x1

|x| < 1 10100 ; x 0

|x| < 1 10100

x!
nPr
nCr
Pol(x, y)
Rec(r, )

o' ''

o' ''

0<
=x <
= 69 (x la so nguyen)
10
0<
= n < 1 10 , 0 <
=r <
= n (n, r la so nguyen)
100
1<
= n! (nr)! < 1 10

10
0<
= n < 1 10 , 0 <
=r <
= n (n, r la so nguyen)
100
100
1<
= n!/r! < 1 10 hay 1 <
= n! (nr)! < 1 10

x, x

99
<
= 9,999999999 10

99
x2 y2 <
= 9,999999999 10
99
0<
=r <
= 9,999999999 10

: nh sinx
|a|, b, c < 1 10100
0<
= b, c
Hien th gia tr giay la chu e sai so 1 tai v tr thap
phan th hai.
|x| < 1 10100
Chuyen oi thap phan he o, phut giay
o
0o00 <
= |x| <
= 9999999 5959
x > 0: 1 10100 < ylogx < 100
x = 0: y > 0
37

x < 0: y = n,

m
(m, n la so nguyen)
2n 1

Tuy nhien: 1 10100 < ylog |x| < 100

a bc

y > 0: x 0, 1 10100 < 1/x logy < 100


y = 0:x > 0
2n 1
y < 0:x = 2n + 1,
( m 0; m, n la so nguyen)
m
Tuy nhien: 1 10100 < 1/x log |y| < 100
Toan bo so nguyen, t so va mau so phai la 10 ch
so hay t hn (ke ca dau chia)

Ranlnt# (a, b) a< b; |a|, |b| < 1 1010 ; b a < 1 1010

Loi

o chnh xac ve can ban la nh a mo ta tai Mien tnh toan va o


chnh xac tren.
Cac ham kieu xy, x y , 3 , x!, nPr, nCr oi hoi tnh toan ben trong
lien tiep, ieu co the gay ta tch luy sai so thng xuat hien cho
tng tnh toan.
Sai so c tch luy co xu hng ln len trong lan can cua iem k
d va iem bung phat cua ham.

May tnh tay se hien th thong bao loi bat k khi nao loi xuat hien bi
bat k l do nao trong qua trnh tnh toan. Co hai cach e ra khoi hien
va
e hien th v tr cua loi, hay nhan
th thong bao loi: Nhan
e xoa thong bao va tnh toan.

Hien th v tr loi

Trong khi thong bao loi ang c hien th, nhan


va
e tr
ve man hnh tnh toan. Con tro se c nh v tai v tr ni loi xuat
hien, san sang cho viec a vao. Hay lam nhng sa cha can thiet
cho tnh toan va thc hien lai no.
Khi ban a nham vao 14 0 2 = thay v 14 10 2 =
14

(hay

38

Xoa thong bao loi


Khi thong bao loi c hien th, nhan
e tr ve man hnh tnh
toan. Lu y rang ieu nay cung xoa tnh toan co cha loi.

Thong bao loi


Math ERROR
Nguyen nhan: Ket qua trung gian hay cuoi cung cua tnh toan ban
ang thc hien vt qua mien tnh toan cho phep. D lieu cua ban
vt qua mien d lieu cho phep (ac biet khi dung cac ham). Tnh
toan ban ang thc hien khong hp le (nh chia cho khong).
Hanh ong: Kiem tra cac gia tr vao, giam bt so ch so, va th
lai. Khi dung bo nh oc lap hay bien lam oi cho ham, hay chac
chan rang bo nh hay gia tr bien ben trong mien cho phep oi vi
ham o.
Stack ERROR
Nguyen nhan: Tnh toan ban ang thc hien a gay ra kha nang
nhom so hay chong lenh b vt qua.
Hanh ong: n gian hoa bieu thc tnh toan e cho no khong
vt qua kha nang cua chong. Th chia tnh toan thanh hai hay
nhieu phan.
Syntax ERROR
Nguyen nhan: Co van e vi dang thc cua tnh toan ban ang thc
hien.
Hanh ong: Hay xem lai quy tac tnh toan.
Loi Insufficient MEM
Nguyen nhan: Cau hnh cua tham bien phng thc TABLE lam
phat sinh hn 30 gia tr -X can c sinh ra cho bang.
Hanh ong: Thu hep mien tnh toan cua bang bang cach thay oi
Start, End va nhng gia tr Step, roi th lai lan na.

Trc khi xac nh may tnh tay truc trac


Hay thc hien cac bc sau bat k khi nao loi xuat hien trong tnh toan
hay khi ket qua tnh toan khong phai la ieu ban trong i. Neu mot
bc khong sa c van e, hay chuyen sang bc tiep.
Lu y rang ban phai lam cac ban sao tach rieng cua d lieu quan
trong trc khi thc hien cac bc nay.
1. Kiem tra bieu thc tnh toan e am bao rang no khong cha
loi nao.
2. am bao rang ban ang dung ung phng thc cho kieu tnh toan
39

ban ang th thc hien.


3. Neu cac bc tren khong sa c van e mac phai, nhan phm
. ieu nay se lam cho may tnh tay thc hien mot trnh kiem tra
lieu ham tnh toan co van hanh ung khong. Neu may tnh tay phat
hien ra bat k bat thng nao, no t ong khi au lai phng thc
tnh toan va xoa noi dung bo nh. Chi tiet ve thiet at c khi
au, xem trong Lap cau hnh thiet at may tnh tay.
4. Khi au tat ca cac phng thc va thiet at bang viec thc hien
(CLR)
(Setup)
(Yes).
thao tac sau:

Thay the pin


Pin yeu c ch ra bi hien th m, cho du a ieu chnh o tng
phan, hay c ch ra bi viec khong hien hnh tren man hien th ngay
sau khi ban bat may tnh tay. Neu ieu nay xay ra, hay thay pin bang
pin mi.
ieu quan trong: Viec thao pin ra se lam cho tat ca noi dung bo nh
cua may tnh tay b xoa het.
1. Nhan

(OFF) e tat may tnh tay.

2. Thao vo boc nh c ve trong minh hoa va


thay pin, can than lap ung cc dng (+) va
cc am ().
3. Lap lai vo.
4. Khi ong lai may tnh:
(CLR)

(All)

(Yes)

ng bo qua bc tren!

ac ta
Yeu cau nguon: pin c AAA R03 (UM-4) 1
Tuoi tho pin xap x: 17 000 gi (hien th lien tuc vi con tro nhap
nhay)
Tieu thu nguon: 0,0002 W
Nhiet o van hanh: 0oC cho ti 40oC
Kch thc: 13,8 (C) 80 (R) 162 (D) mm
Trong lng xap x: 100 g ke ca pin
40

Cau hoi thng gap


Lam sao toi co the thc hien a vao va hien th ket qua theo cung
cach toi a lam tren mo en khong co Hien th sach t nhien?
Hay thc hien thao tac phm sau
(SETUP)
(LinelO). Xem
cai at cho may e biet them thong tin.
Lam sao toi co the thay oi c dang thc phan so sang dang thc
thap phan? Lam sao toi co the thay oi ket qua dang thc phan so
c tao ra bi phep chia cho dang thc thap phan?
Xem muc Chuyen ket qua tnh toan e biet them thong tin.
Khac biet la the nao gia bo nh Ans, bo nh oc lap va bien bo
nh?
Moi mot trong cac kieu bo nh eu hanh ong nh bnh cha cho viec
lu gi tam thi mot gia tr.
Bo nh Ans: Lu tr ket qua cua tnh toan cuoi cung c thc hien.
Dung bo nh nay e mang ket qua cua tnh toan nay sang tnh toan
khac.
Bo nh oc lap: Dung bo nh nay e cha tong cac ket qua cua nhieu
phep tnh toan.
Bien: Bo nh nay co ch khi ban can dung cung mot gia tr nhieu lan
trong mot hay nhieu tnh toan.
Thao tac phm nao a toi t phng thc STAT hay TABLE sang
phng thc toi co the thc hien tnh toan so hoc?
Nhan
(COMP).
Lam sao toi co the a may tnh tay tr ve cai at mac nh khi au
cua no?
Hay thc hien cac thao tac sau:
(CLR)
(Setup)
(Yes)
Khi toi thc hien mot tnh toan ham, tai sao toi thu c ket qua tnh
toan hoan toan khac vi cac mo en may tnh tay CASIO cu hn?
Vi mo en Hien th sach t nhien, oi cua ham dung cac dau ngoac tron
phai co theo sau mot dau ngoac ong. Khong nhan
sau oi e
ong dau ngoac lai co the tao ra nhng gia tr khong mong muon c
a vao nh mot phan cua oi.
V du: (sin 30) + 15
Mo en S-VPAM cu:

30
30

Mo en Hien th sach t nhien:


Khong nhan

15

15.5

15

15.5

ay nh c neu di ay se tao ra tnh toan cua sin 45.

30

41

15

0.7071067812

GIAI CAC BAI TOAN THUOC


CHNG TRNH TRUNG HOC C S
LP 6
I.

So t nhien:
1.

Tap hp cac so t nhien:


Liet ke so lien sau, lien trc cua mot so (dung phm
).
V du:
Tm cac so lien sau, lien trc cua 60
Giai
1 e tm so lien sau. Sau o, ta ch viec bam
se
An 60
se c so lien sau ke tiep, ...
c so lien sau, tiep tuc bam
1 e tm so lien trc. Sau o, ta ch viec bam
se
An 60
c so lien trc, tiep tuc bam
se c so lien trc ke tiep, ...

2.

Phep cong va phep nhan


S dung cac phm tnh toan c ban (lu y ve gii han tnh toan cua
may va th t u tien cua phep toan)
Dung may tnh e tnh :
V du 1:
a) 2314 + 359
b) 2374 + 359
c) 2374 + 39
d) 2374 + 379
Giai:
a) An e ghi len man hnh
2314
359 va an
Ket qua 2673
e a con tro len dong bieu thc va dung phm
b) An
359 va an
Ket qua 2733
chnh lai thanh 2374
c) An
e a con tro len dong bieu thc va dung phm
39 va an
Ket qua 2413
chnh lai thanh 2374
e a con tro len dong bieu thc va chnh lai thanh
d) An
2374 + 379 va an
Ket qua 2753
Ghi chu : May ch oc c mot so co 15 ch so, neu ghi dai hn
na may khong hieu
V du :
An 1234567893456789
1234567891234567 va an
May hien ket qua sai la 2222220 v may khong oc c ch so
th 16
42

V du 2: Tnh
345 + 45 + 7652 + 56
Giai
Nhap vao man hnh
45
7652
345

56 va an

Ket qua:8098

V du 3: Tnh:
a) 269 38
b) 64 986
c) 76 (456+87)
d) (79 + 562) 94
e) (54 + 27) (803 +27)
f) 34 + 38 76 + 548 7 +79
Giai
C ghi y het tng bieu thc tren vao man hnh va an
se c
ket qua. May Casio 500VN PLUS (va tat ca cac loai may tnh
khoa hoc khac) la may tnh co u tien nen cach tnh khac han cach
tnh cua may n gian (loai ch co phm +, - , , , %,
S:
a) 10222
b) 63104
c) 41268

, ...)

d) 60254
e) 67230
f) 6837

V du 4: Khi an 1
2
3
th may n gian cho ket qua la
9 (may nay tnh 1 + 2 = 3 sau o tnh 3 3 = 9 ngha la an en
au may tnh ngay en ay)
Trong khi ay may tnh khoa hoc (co may Casio 500VN PLUS)
cho ket qua la 7
(may oc ca bieu thc roi ap dung th t u tien cac phep tnh
ung nh thay day lp hoc. Phep nhan, chia u tien hn phep
cong tr nen tnh trc 2 3 = 6 roi mi tnh tiep 1 + 6 = 7).
Rieng dau nhan lien trc dau ngoac th co the bo qua
76 (456+87) co the ch ghi 76 (456+87)
(xin xem them ghi chu phan phep chia va phep nhan trong cung
mot bieu thc tiep sau).
Dau ong ngoac cuoi cung (se an tiep
e tm ket qua) cung
co the khoi an.
Bai (54 + 27) (803 +27) = (54 + 27) (803 +27)
= (54 + 27) (803 +27) = 67230.
c may tnh giong het sach giao khoa
43

Bai 34 + 38 76 + 548 7 +79 = 6837 c may tnh giong het


sach giao khoa (phep nhan u tien hn phep cong).
Ghi chu: Khi gap phep nhan co ket qua qua 10 ch so va di 15
ch so ma neu e lai yeu cau ghi ay u, v du nh :
8567899 654787
Ta co the lam nh sau:
Nhap vao bieu thc tren an
ta thay ket qua

5,61014888310 12
5,6101
12
(48882513)
An tiep
Vay ket qua ay u la: 561014882513
Bai tap thc hanh:
1) Tnh cac tong sau:
a. 1364 + 4578
b. 31214 + 1469
ap so:
a. 5942
b. 32683

c. 7243 + 1506
d. 1534 + 231 + 4056 + 4690
c. 8749
d. 10511

2) Tnh:
a. 21 (649 + 123)
b. -21 649 +123
ap so:
a. 16212
b. -13506

c. (54 +16) (812 +12)


d. 8585869 9043
c. 57680
d. 77642013367

3) Tm x , biet:
a. (x-27) 2 = 108
b. 3x (28+32) = 6
ap so:
a. 243
b. 120

c. 19 x (4x-21) = 0
d. 943 (x+3) = 41
c. 5,25
d. 20

4) Nam abcd Tran Hng ao viet Hch Tng S khuyen ran cac
tng s chuan b cho cuoc khang chien chong quan Nguyen
xam lc lan th 2. Biet rang ab la tong so thang trong mot
nam, con cd gap 7 lan ab . Tnh xem nam abcd la nam nao ?
ap so: 1284
Tnh toan giai tha
N: n! = 1. 2. 3 ... n
V du:
Tnh
44

a. 6!
b. 7!-9!
a. An 6
b. An 7
3.

(x!)
(x!)

Giai
720
9

(x!)

-357840

Phep tr va phep chia (kha tng t 2 phep toan tren)


V du 1 : Tnh
a) 269 38
b) 552 12
c) (1602 785) 19
d) 45591 (318 45)
e) (49407 3816) (318 45)
f) 315 387 9 476 17 59
Giai
C ghi y het tng bieu thc tren vao man hnh va an
se c
e tm ket
ket qua. Dau ong ngoac cuoi cung (se an tiep
qua) cung co the khoi an.
V du : Bai

45591 (318 45) co the ch ghi


45591 (318 45 va an

S:
a) 231
b) 46
c) (1602 785) 19 = 43
d) 45591 (318 45) = 167
e) (49407 3816) (318 45) = 167
f) 315 387 9 476 17 59 = 185
c may tnh giong het sach giao khoa (phep chia u tien hn
phep tr)
Bai tap thc hanh
1) Tnh
a) 8072 5769
b) (3472 3081) 17
c) 6034 (306 + 125)
d) (9875 6540) (2682 2015)
2) Tm x , biet
a) 17x 595 = 1581
b) 380 (2x + 75) = 105
c) (6x12) 12 = 828
d) 1206 (2x+3) = 18
45

S: 2303
S: 23
S: 14
S: 5
S: 128
S: 100
S: 1658
S: 32

4.

Phep tnh hon hp


V du 1: Tnh
a) (49407 - 3816) (114 + 53)
b) 315 - 387 9 + 476 17 59
Giai
se c
C ghi y het tng bieu thc tren vao man hnh va an
ket qua
a) (49407 - 3816) (114 + 53) = 273
b) 315 -387 9 + 476 17 59 = 1924
(Khi khong co dau ngoac th phep nhan, chia u tien hn phep
cong, tr)
phan 2 co noi dau nhan lien trc dau ngoac th co the bo qua.
V du 2 : 76 (456+87) co the ch ghi 76 (456+87)
Nhng phai phan biet rang:
Phep nhan tat u tien hn phep nhan thng do o phep nhan
tat u tien hn phep chia.
Ta hay xet v du sau
Neu ghi 36 3 (4 + 2) va an
Ket qua la 72
Neu ghi 36 3 (4 + 2) va an
Ket qua la 2
Cung vay 36 3 4 hoan toan khac vi 36 3 (4
Do 3(4+2) va 3(4 la phep nhan tat nen u tien hn phep chia
Quy nh nay ch ap dung vi may 500VN PLUS, Casio-500MS,
va cac may ho MS .
Vi cac may ho khac th phai theo hng dan cua may ho ay
Bai tap thc hanh
a) (145624 9872) (197 + 371)
S : 239
b) 405 564 12 + 21 78 18
S : 449
c) (512 137) (3567 29) (70423) (243+109)+217
S : 46296
d) (203 560 16 (3609+3491) 25) 19 .
S :359

5.

Luy tha vi so mu t nhien:


a) Tnh gia tr cua mot luy tha
V du:
Tnh 25, 37, 52, 73
Tm 25 : An 2
5
32
46

37 : An 3
7
2187
Lam tng t ta co:52 =25, 73 = 343
b) Tnh toan n gian tren luy tha(cong, tr, nhan, chia..)
V du:
Tnh
1 ) 25+52 5-33
2) 24.12-26.9
3) 40 [25 (4 1)2]
An nh sau:
1) 2
5
5
5
3
(10)
2) 2
4
12
2
6
9
(-384)
25
4
1
(24)
3) 40
c) Tm ch so tan cung cua 1 luy tha
V du 1: Tm ch so cuoi cua 7 2005
Giai
Ta khong the dung may e tnh trc tiep c ma phai theo giai
thuat sau
71 = 7

7 6 = 117649

7 2 = 49

7 7 = 823543

7 3 = 343

7 8 = 5764801

7 4 = 2401

7 9 = 40353607

7 5 = 16807

.....

Bang tren co the tao de dang ma khong can phai tnh toan vi chc
nang TABLE cua may nh sau:
+ Khi ong che o TABLE:
+ Tren may se hien f(X) chung ta nhap vao ham
(x)(do ay la luy tha cua 7)
7
(Start?)1
(End?)9
(Step?)1
+ An tiep :
(Start la gia tr bat au, ta nhap la 1 (do bat au la 71),
ket thuc la 9(do gia tr cuoi la 79)
Theo tren, ta thay cac so cuoi lan lt la 7 , 9, 3, 1 chu k la 4
Mat khac 2005 = 4 501 + 1 7 2005 co so cuoi la 7
V du 2
Tm ch so tan cung cung cua 4 2008
Giai
Ta cung dung chc nang TABLE :
. Nhap ham 4
(x) .
An
Roi an tiep
(Start?)1
(End?)9
(Step?)1
47

(Start la gia tr bat au ,ta nhap la 1 (do bat au la 41), ket thuc la
9 (do gia tr cuoi la 49)
Sau khi thc hien xong ta c bang sau:
41 = 4

4 6 = 4096

4 2 = 16

4 7 = 16384

4 3 = 64

4 8 = 65536

4 4 = 256

4 9 = 262144

.......................
4 5 = 1024
Theo bang tren, ta de thay cac so cuoi lan lt la 4,6 chu k la 2.
Ma 2008 = 2 1004 42008 co so cuoi la 6
d) Phep ong d
Khi a chia b c d la c, ta viet: a c(mod b)
Ta co tnh chat sau:
a c(mod b)
a b c d(mod b)

b d(mod b)

Chung ta ng dung tnh chat nay trong viec tm so d cua phep


chia vi so b chia c cho bang dang luy tha qua ln
V du 1: Tm so d cua phep chia 14 7 cho 23

14 1 14(mod 23)
14 2 12(mod 23)
14 4 (12) 2 6(mod 23)
14 7 14 12 6 19(mod 23)
Ket qua so d la:19
V du 2 : Tm so d cua phep chia 2008 324 cho 1986
V 324 = 108 3

2008 2 484(mod 1986)


2008 4 484 2 1984(mod 1986)
2008 12 1894 3 1810(mod 1986)
2008 36 1810 3 1780(mod 1986)
2008 108 (1780) 3 556(mod 1986)
2008 324 (2008 108 ) 3 (556) 3 1246(mod 1986)
Chu y : chung ta khong tnh thang so d cua 20084 chia cho 1986
c v ay phep tnh so d cua phep chia 20084 cho 1986 rat
de b hieu lam do neu ghi 20084 1986 va an
may hien
8186072558 khien ta tng o la so nguyen , thc ra so ay la
8186072557.95
48

Do vay khi s dung may tnh ma gap ket qua la 1 so nguyen va


u 10 ch so th ta phai canh giac rang o co the khong phai la
mot so nguyen ma ch la 1 so thap phan b lam tron
e) Tm ch so hang chuc cua luy tha
Ta ch can dung ong d mod 100
V du: Tm ch so hang chuc cua 19869
1986 3 56 (mod 100)
1986 9 56 3 16 (mod 100)
Vay ch so hang chuc la 1
f) Tm so mu cua 1 luy tha:
V du : Tm so t nhien n sao cho 2n = 64
Nhap vao man hnh:
2
6.

(x)
(Start?)1
(End?) 9
(Step?)1
May se xuat ra 1 bang tra bang thay x=6 la gia tr can tm

Phep chia co so d:
a) Trng hp so b chia nam trong mien tnh c (<10 ch so)
Nh a biet: So b chia = thng So chia + So d
So d = So b chia - thng So chia
(vi thng ay la phan nguyen trong phep chia lay so thap
phan)
Do o e tm so d cua phep chia ta lam nh sau:
+ Nhap bieu thc va thc hien phep chia
e chuyen ve gia tr thap phan (neu ang
+ Dung dau
che o MATH
+ Ghi nh phan nguyen cua ket qua (phan trc dau
phay(,) hay dau cham(.) )
e chnh lai dau thanh dau - roi
+ Dung dau
chuyen ve cuoi phep tnh an them
roi nhap tiep phan
nguyen a nh, sau bam
Lu y: e de dang thc hien thuat giai nay nen e may che
o
Neu e che o
may van thc hien c
nhng ta phai lam them bc 2, ong thi phai nh gia
tr nguyen, rat phc tap
V du 1:
Tm so d cua phep chia:
1. 134 43
2. 435 98756
49

3. 266 17889
4. 2314 1293

Giai
1. che o
ghi vao man hnh:13
4
Ket qua thu c la:446,265625
Dung dau
them vao cuoi phep tnh
sa dau
thanh dau
.
Ket qua ta thu c 17
Lam tng t cac cau con lai.
ap so:
1. 17
2. 59515
3. 8524
4. 886707

446. Sau o dung

V du 2: Tm so d cua phep chia 55296037 cho 793056


Giai
Ghi vao man hnh: 55296037
793056
Ket qua phep tnh: 69,72526
An
e a con tro len man hnh sa dau
thanh dau
va nhan 69 sau 793056 roi an
Man hnh tr thanh: 55296037
793056
69
575173
V du 3:
Tm so d cua phep chia 7781188255 cho 37568704
Giai
Ghi vao man hnh: 7781188255
37568704
Ket qua tnh: 207,11889
e a con tro len man hnh sa dau
thanh dau
An
va nhan 207 sau 37568704.
37568704
207
Man hnh tr thanh: 7781188255
4466527
Bai tap thc hanh
Tm so d cua cac phep chia sau:
a. 803868110 cho 3686645
b. 563648261 cho 6231202
ap so :
a. 179500
b. 2840081

b) Trng hp so b chia nhieu hn 10 ch so


Trng hp so b chia nhieu hn 10 ch so cat ra thanh nhom
50

au 9 ch so (ke t ben trai) tm so d nh phan 6a. Viet lien tiep


sau so d con lai toi a u 9 ch so roi tm so d lan 2, neu con
na th tnh lien tiep nh vay.
V du:
Tm so d cua phep chia 705369747425611 cho 345
345
Ta tm so d cua 705369747
Ket qua la 342
345
Ta tm so d cua phep chia 342425611
Ket qua la 1
Bai tap thc hanh
1. Hay ien vao o trong

So b chia
So chia
Phan nguyen cua thng
So d

82849
471

2.Tm so d cua phep chia:


a. 9130429 cho 4576
b. 64131261 cho 9485
c. 92549376 cho 4791
d. 93629500 cho 5743

74785
257
19
11
17

26391308
8274

S: 1309
S: 3176
S: 1629
S: 1371

7.

Dau hieu chia het


Bo sung:
+ So nao va chia het cho 2 va chia het cho 3 th chia het cho 6
V du : 582 va chia het cho 2 (tan cung bang so chan) va chia
het cho 3 (co tong 5+8+2=15 chia het cho 3) nen chia het cho 6
+ So nao co hai ch so tan cung hp thanh so chia het cho 4 th
chia het cho 4
V du : 1896 co 2 so tan cung la so 96 chia het cho 4 th chia het
cho 4.
Nam nhuan (thang hai co ngay 29) la nam ma so ghi nam chia
het cho 4, tr nam tron the ky ma so the ky khong chia het cho
4. Hay cho biet cac nam 1600, 1700, 1900, 1991, 1992, 2000 co
nam nao la nam nhuan.
ap so : 1600, 1992, 2000

8.

Boi va c
a) Liet ke boi cua 1 so
Cach 1: Dung vong lap
V du 1: Tm tap hp boi cua 120
51

Cac boi cua 120 la:


120 0 = 0
120 1 = 120
120 2 = 240
120 3 = 360
............
V du 2: Tm tap hp boi cua 30
Cac boi cua 30 la:
30 0 = 0
30 1 = 30
30 2 = 60
30 3 = 90
............
S dung may
(STO)
(A)
Nhap vao may -2
(A)
1
(STO)
(A)
(:)30
(A)
An
ket qua ra 0.
An
tiep se ra cac gia tr boi so tiep theo (moi lan sau 2
mi la boi cua 30, con sau 1 dau
la gia tr cua bien
dau
em)
Cach 2: Dung chc nang TABLE cua may.
Vi cong thc boi cua a la ax , vi x la so nguyen
V du 1: Liet ke 5 phan t cua tap hp boi cua 25
An
(x)
(Start?)0
Nhap vao cong thc boi cua 25: 25
(End?)5
(Step?)1
.
Cot th 3 ben phai man hnh chnh la cac boi cua 25
Lu y: cach 2 nhanh hn cach 1 v khong can bam phm bang .
Nhng han che la ch tnh c 30 boi so au tien
Mot so v du ve boi
V du 2: Tm cac boi so nho hn 2006 cua 206
Giai
An
(x)
Nhap vao cong thc boi cua 206:206
(Start?)0
(End?)10
(Step?)1
(ta cho gia tr cuoi la
10 v 2060 > 2006)

52

May xuat ra bang nh sau:


x
1
0
2
1
3
2
4
3
5
4
6
5
7
6
8
7
9
8
10
9
11
10

F(x)
0
206
412
618
824
1030
1236
1442
1648
1854
2060

Ket qua boi cua 206 nho hn 2006 la : 0, 206, 412, 618, 824,
1030, 1236, 1442, 1648, 1854.
V du 3: Tm boi cua 45 nho hn 2000 va chia het cho 35
Giai
An - 2
(STO)
A
(A)
1
(STO)
(A)
An
e a con tro ve cuoi dong bieu thc ben phai, an tiep
(:)(hai cham mau tm) 45
(A)
35
(:)45
(A)
An
Man hnh hien 0
0
0
Ngha la 45 0 35 = 0 va 45 0 = 0, do 0 35 = 0 suy ra 0 chia
het cho 35. Vay ta nhan 0.
Tiep tuc an
va e y neu thay man hnh hien 45A
35 la so
ke tiep chnh la so
nguyen th so nguyen hien ra trong lan an
thoa ieu kien bai toan
man hnh hien 7
9
315
Ta e y thay khi an
Khi o 315 la so can tm, tiep tuc an nh the ta tm c 5 so na
thoa ieu kien bai toan la :630, 945, 1260, 1575, 1890 .
Khi thay ket qua ln hn 2000 th ngng an .
S: 0,315 , 630 , 945 , 1260 , 1575 ,1890 .
Bai tap thc hanh
1) Tm boi cua 103 nho hn 1000 .
S: 0, 103, 206, 309, 412, 515, 618, 721, 824, 927
2) Tm boi cua 215 ln hn 1000 va nho hn 2000 .
S : 1075 ,1290 , 1505 , 1720 , 1935.
53

3) Tm boi cua 32 chia het cho 48 , ln hn 500 va nho hn 800.


S: 576 , 672 , 768 .
b) Liet ke c cua 1 so
V du: Tm c cua 120
Cach 1:
An 0

An

(STO)
(A)
(A)
1
(STO)
(A)
e a con tro ve cuoi dong bieu thc ben phai ,

An tiep
(:) 120
(A)
Ta ch lay ket qua la so nguyen
An
Man hnh hien 2
Ket qua : 60 (co ngha la 120 2)
Man hnh hien 3
An
Ket qua : 40 (co ngha la 120 3)
An
Man hnh hien 4
Ket qua : 30 (co ngha la 120 4)
Man hnh hien 5
An
Ket qua : 24 (co ngha la 120 5)
Man hnh hien 6
An
Ket qua : 20 (co ngha la 120 6)
An
Man hnh hien 7
120
Ket qua :
(co ngha la 120 7)
7
An
Man hnh hien 8
Ket qua : 15 (co ngha la 120 8)
Man hnh hien 9
An
40
Ket qua :
(co ngha la 120 9)
3
An
Man hnh hien 10
Ket qua : 12 (co ngha la 120 10)
Man hnh hien 11
An
120
(co ngha la 120 11)
Ket qua :
11
120
11 nen ngng an
Ta thay
11
Ket qua
U (120) = {1, 2, 3, 4, 5, 6, 8, 10, 12, 15, 20, 24, 30, 40, 60, 120}
vi 1;120 la 2 c so tam thng cua 120
54

Cach 2 :
Tng t phan b tren dung chc nang TABLE cua may.
Ta biet 1 so bat ky luon co 2 c tam thng la 1 va chnh no. Do
o ta ch can tm cac c con lai.Vi cong thc c cua a la :
a x,vi x la so nguyen dng ln hn 2 va nho hn phan
nguyen cua can x cong 1
V du: Liet ke c cua 300

An

300

17,32

Nhap vao cong thc ccua 300: 300


(x)
(Start?)2
(End?)18
(Step?)1
.
Ta xem cot ben phai cua bang au nguyen th gia tr tng ng
cot au tien ben trai la c cua 300 va nh ghi them gia tr cuoi
la 300
Ket qua: c cua 300 la1; 2; 3; 4; 5; 6; 10; 12; 15; 20; 25; 30; 50;
60; 75; 100; 150; 300.
Bai thc hanh
Tm c cua cac so sau
a) 48
d) 308
b) 52
e) 1980
c) 310
f) 7890
9.

So nguyen to
V du: So 647 co phai la so nguyen to khong ?
Giai
Cach 1: Chia 647 cho cac so nguyen to 2, 3, 5, 7, 11, 13, 17, 19,
23, 29, (ket hp chia tren may va nhan nh cac dau hieu chia het).
Khi chia cho 29 th thng la 22, 3 . . . < 29 nen ngng chia va ket
luan 647 la so nguyen to.
Cach 2: Kiem tra mot so co phai la so nguyen to hay khong, bang
phng phap lap. au tien chnh may che o
An 1
(STO)
(x) (Gan so 1 cho x)
An tiep
(x)
2
(STO)
(x)
(:) 647
(x)
647
An
(3)
c ket qua la 215,(6)

An
lien tuc cho en khi x ln hn thng th dng(trong qua trnh
bam them
e hien gia tr thap phan),
bam
Trong qua trnh an phm bang neu thng tra ve gia tr nguyen th
55

dng ngay va ket luan so a cho khong phai la so nguyen to.Ngc


lai neu toan bo qua trnh eu cho thng la so thap phan th ket
luan so a cho la nguyen to.
Cach 3: Dung chc nang TABLE cua may
Kiem xem a co phai nguyen to khong.
Ta dung cong thc sau e kiem tra: a:x,
vi x la so nguyen thoa 3 x 61
Neu ket qua cot th 3 ben phai co gia tr la so nguyen th ta ket luan
a la hp so, ngc lai a la nguyen to.
Lu y: Cach 3 li ve thi gian bam phm, chung ta khong phai an
dau bang va ch nh cach 2 .Nhng no ch kiem c nhng so
nguyen dng nho hn 3721.
V du: Kiem xem cac so sau so nao la nguyen to.
a. 859
b. 417
c. 900
d. 1249
Giai
a.
S dung cach 2: au tien chnh may che o
(STO)
(x) (Gan so -1 cho x)
An 1
An tiep
(x)
2
(STO)
(x)
(:) 859
(x)
An
c ket qua la 286,(3)
lien tuc cho en khi x bang 31 th c thng la
An
bam them
e
27,70967742 th ta dng (trong qua trnh bam
hien gia tr thap phan)
Trong suot qua trnh bam khong thay thng co gia tr nguyen. Vay
ket luan 859 la so nguyen to.
S dung cach 3:
De thay 859 khong chia het cho 2 do ch so tan cung la le
Ap dung thuat toan tren
An
(x)
Nhap vao cong thc:859
ay ta co the nhap x thoa ieu kien goc 3 x 61.Tc la nhap
vao man hnh nh sau:
(Start?)3
(End?)61
(Step?)2
.
May se xuat ra bang,nhn nhanh qua cot th 3 ben phai ta thay
khong co gia tr nao nguyen.Do vay ta ket luan 859 la so nguyen to.
Tuy nhien cach nhap x tren cha phai toi u v x khong can phai ti
56

61. Ta co the thu gon mien gia tr cua x bang cach lay xmax= phan
nguyen cua can x cong 1 ay

x = 29,309.Ta lay xmax= 30.

Do o ta nhap vao man hnh:


(Start?)3
(End?)30
(Step?)2
.
Bc kiem tra lam hoan toan tng t tren.
b.
De thay 417 khong chia het cho 2 do ch so tan cung la le
Ap dung thuat toan tren :
An
(x)
Nhap vao cong thc:417
ay ta co the nhap x thoa ieu kien goc 3 x 61.Tc la nhap
vao man hnh nh sau:
(Start?)3
(End?)61
(Step?)2
.
Ta thay dong co x=3 gia tr cot th 3 bang 139 (nguyen)
Vay 417 la hp so.
S: c. Hp so
d. Nguyen to
10. Phan tch 1 so ra tha so nguyen to
V du: Phan tch 1800 ra tha so nguyen to
Giai
Ghi vao man hnh :1800
2 va an
.
Ghi tha so 2
Thay ket qua la 900 con chia het cho 2 , nen ghi tiep vao man hnh:
2
Ghi tha so 2.
Thay ket qua la 450 con chia het cho 2 , nen an
Ghi tha so 2.
Thay ket qua la 225 khong chia het cho 2 ma lai chia het cho 3, nen
va chnh man hnh thanh
3 an
an
Ghi tha so 3.
.Ghi tha so 3
Thay ket qua la 75 con chia het cho 3 , nen an
Thay ket qua la 25 khong chia het cho 3 ma lai chia het cho 5, nen
va chnh man hnh thanh
5 va an
Ghi tha so 5
an
Ghi tha so 5
Thay ket qua la 5 con chia het cho 5 , nen an

Ket qua: 1800 = 2 3 3 2 5 2 .


Bai tap thc hanh
Phan tch cac so sau ra tha so nguyen to 150, 1020, 700, 4620,
41580 .
11. c so chung ln nhat va c chung
Ap dung cong thc (a, b) = (a b, b) vi a > b.
57

V du 1: Tm c so chung ln nhat cua 75 va 60


Ta viet:
75-60=15
60-15=45
45 -15=30
30 -15=15
15-15=0
USCLN(75,30) = 15
V du 2: Tm c chung ln nhat cua 75 va 30
Ta viet:
75-30=45
45-30=15
30-15=15
15-15=0
USCLN(75,30) = 15
Cach s dung may e tm USCLN
(A)
75
30
(B)
Nhap vao man hnh
|AB| A:|BA| B
nhieu lan cho en khi xuat hien so 0, ta bam
Sau o bam
them lan na th may a ra mot so khac 0. Vay USCLN (75,30) = 15
Sau khi co c USCLN,muon tm c chung ta ch viec tm c cua
USCLN (cach lam y nh 8b)
12. Boi so chung nho nhat
Can tm boi chung nho nhat cua A va B.
Ta ap dung thuat toan tren, tm USCLN cua A va B.
Roi ap dung cong thc tm BSCNN
AB
BSCNN(A,B) =
USCLN(A, B)

Sau bai n gian phan so ta con co cach tm USCLN, BSCNN khac


na
V du 1: Tm BSCNN cua 75 va 60
S dung ket qua tren, ta thay: USCLN(75,60) = 15
75 60
= 300
BSCNN(75,60) =
15
Vay boi so chung nho nhat la : 300
Lu y: Sau bai phan so chung ta se biet them 1 phng phap tm
c chung ln nhat va boi chung nho nhat nhanh hn rat nhieu.

58

II. So nguyen:
1. Lam quen vi so nguyen am-Tap hp so nguyen:
a. Cach viet 1 so nguyen am tren may tnh:
e bieu dien mot so nguyen am ta ch can them dau
1 so nguyen bat ky
V du 1: An
3
(-3) oc la tr 3 (am ba)
V du 2: An
7
(-7) oc la tru bay (am bay)

vao trc

b. Tm so oi cua 1 so :
V du: Tm so oi cua
a. 7
e. -13
b. -20
f. 23
c. 15
g. -40
d. -25
h. 19
a. An 7
b. An -20
ap so:
c. -15
d. 25
e. 13
2.

Giai
(-7).Man hnh hien ket qua la -7.
(20).Man hnh hien ket qua la 20.

f. -23
g. 40
h. -19

Th t trong Z
a. So sanh 2 so nguyen
Ta lay 2 so can so sanh tr nhau neu ra so am (co dau tr) th so b
tr nho hn so tr, ngc lai neu ra so dng (khong co dau tr) th
so b tr ln hn so tr.
V du:

?4
An may

9;

-5
-8;
5
9
(-5) vay 4 < 9
4
-8
(3) vay -5>-8
-5
Tng t, ta co: 5 > -9; 6 > -6

-9;

-6

b. Tm x gii han trong 1 khoang:


V du: Tm x Z:
1) -5 < x < 0
3) -3 x < 1
2) -4 < x < 2
4) -2 < x 3
Giai
1) An -5
1
(-4)
(-3)
(-2)
(-1)
(0)
Vay x la :-4;-3;-2;-1 (khong lay 0 do khong co dau bang)
1
(-3)
(-2)
(-1)
(0)
(1)
(2)
2) An -4
Vay x la :-3;-2;-1;0;1 (khong lay 2 do khong co dau bang)
59

3) An -3
1
(-2)
(-1)
(0)
(1)
Vay x la:-3(do co dau bang au);-2;-1;0(khong lay 1 do khong
co dau bang cuoi)
4) An -2
1
(-1)
(0)
(1)
(2)
(3)
Vay x la:-1;0;1;2;3(lay 3 do co dau bang cuoi)
c. Tnh toan tren tr tuyet oi
Dung phm
e tnh gia tr tuyet oi
V du: Tnh
d. 5 3
a. 7

b. 7

e. 5 . 3

c. 5 2

f. 6 3
Giai

a. che o
An
7
(7)
che o
An
7
(7)
b. che o
7
(7)
An
che
7
(7)
An
c. che o
5
2
An
che o
5
2
An
Cac cau con lai lam tng t.
3.

(3)
(3)

Cac phep tnh tren so nguyen


V du 1: Tnh:
a)
(+475) + (+2345) + (+7643)
b)
(7654) + (678) + (3167)
c)
(4328) + (+975)
d)
(+7653) + ( 674) + (+32) + ( 428)
Giai
Ghi vao man hnh nh sau va an
sau moi bieu thc (vi dau la
phm
con so am pha trc la dau
)
a)
475 + 2345 + 7643
(S : 10463)
b)
7654 + (678) + (3167)
(S : 11499)
hay 7654 678 3167
60

c)
d)

4328 + 975
7653 674 + 32 428

(S : 3353)
(S : 6583)

V du 2: Tnh
a)
4568 (+671)
b)
(+876) (345)
c)
( 43267) + (+123) (+598) ( 4179)
d)
567 + 8764 3456 + 45 28
Giai
Ghi vao man hnh nh sau va an
sau moi bieu thc (vi dau la
phm
con so am pha trc la dau
)
a)
4568 671
(S : 3897)
b)
876 + 345
(S : 1221)
c)
43267 + 123 598 + 4179
(S : 39563)
d)
567 + 8764 3456 + 45 28
(S: 5892)
V du 3: Tnh
324 + 841 [112 (35 +79)] + 41
Giai
Ghi vao man hnh nh sau va an
324 + (841 (112 (35 +79))) + 41
(KQ : 1208)
V du 4: Tnh
a)
(+ 456)
b)
(+243)
c)
(123)
d)
(321)

(+8962)
(547)
(+712)
(345)

Giai
Ghi vao man hnh nh sau va an = sau moi bieu thc
a)
456 8962
(S : 4086672)
b)
243 547 dau am () trc 547ghi bang phm
(S : 132921)
c)
123 712 dau am () trc123 ghi bang phm
(S :87576)
d)
321 345 dau am () trc 345 ghi bang phm
(S : 110745)
V du 5: Tnh
a)
(+ 456) [(+476)( 94)]
b)
[( 38) + (875) (+65)] [(67) + 239]
c)
(781123) 278
Giai
Ghi vao man hnh nh sau va an
sau moi bieu thc
61

a) 456
b)
c)

476
38
875
781

123

(S : 259920)
67
239
(S : 168216)
(S : 182924)

94
65
278

III. Phan so:


1. M rong khai niem phan so:
a. Cach nhap phan so:
e nhap phan so
Dung phm
a
, ta an a
b
Muon nhap phan so
b
b. oi phep chia ra phan so:
Ta nhap phep chia bnh thng, bam
may se t chuyen
c. oi so thap phan ra phan so:
may se t chuyen
Ta nhap phan so bnh thng, bam
2.

Phan so bang nhau:


a. S dung chc nang RATIO cua may
V du 1: Tm x, biet:
x
6
=
1.
15
18
8
24
=
2.
x
18
Giai
1. An
(Ratio) 1(a:b=x:d)
Nhap cac he so:(a)6
(b) -18
(Ratio) 2(a:b=c:x)
2. An
Nhap cac he so: (a)24
(b)18

(d)15
(c) 8

(x=-5)
(x=6)

V du 2: Kiem tra xem cac phan so sau co bang nhau khong:


4
16
,
a.
7
28
21 28
,
b.
12
15
Giai
a. An
(Ratio) 1(a:b=x:d)
Nhap cac he so:(a)-4
(b) 7
(d)28
(-16)
Vay 2 phan so bang nhau
(Ratio) 1(a:b=x:d)
b. An
(b)12
(d)15
(-26.25)
Nhap cac he so:(a)-21
Do -26.25 -28 nen 2 phan so khong bang nhau.
62

Bai tap thc hanh :


1. Tm x,y,z biet :
15
5
x
z
=
=
=
6
12
24
y

An
(Ratio) 1(a:b=x:d)
Nhap cac he so:(a)-5
(b) 6

(d)12

(-10) x=-10

(Ratio) 2(a:b=c:x)
An
(b) 6
Nhap cac he so:(a)-5

(c)-15

(18) x=18

An
(Ratio) 1(a:b=x:d)
Nhap cac he so:(a)-5
(b) 6

(d)-24

(20) x=20

2. Tm x biet:
3
x
a.
=
4
20
4
12
=
5
x
ap so:
a. x = 15
b. x = 15

b.

c.

x
16
=
9
36

d.

7
21
=
x
39

c. x = -4
d. x = -13

b. Viet phan so di dang phan so co mau so dng


5
6
19
V du: Viet cac phan so sau
,
,
28 13 37
di dang phan so co mau dng :
Giai
19
An
19
28

28

3.

13

13

37

37

Rut gon phan so:


V du:

Rut gon phan so


Ghi vao man hnh
13
Ket qua
19

221
323

221

323 va an

63

Bai tap thc hanh


1) Rut gon cac phan so sau
30
448
b)
a)
48
840
13
12
149 299
21
d)
e)
392
265
1
42
536
S:
5
8
b)
a)
8
15
7504
d) 2
e)
29
4.

c)

735 215
621 46

c)

38
23

Tm USCLN va BSCNN
Ta ng dung tnh rut gon bieu thc tren e tm USCLN va BSCNN
bang thuat giai sau :
A
= a
(toi gian)
B
b
th USCLN cua A, B la A a
BSCNN cua A, B la A x b
V du 1: Tm USCLN va BSCNN cua 209865 va 283935
283935 va an
Ghi vao man hnh: 209865
17
Man hnh hien :
23

e a con tro len dong bieu thc sa thanh


An
Ket qua : USCLN = 12345
209865 17 va an
An
e a con tro len dong bieu thc sa thanh
209865 23 va an
Ket qua : BSCNN = 4826895
V du 2: Tm USCLN va BSCNN cua 2419580247 va 3802197531
Ghi vao man hnh 2419580247
3802197531 va an
7
Man hnh hien
11

e a con tro len dong bieu thc sa thanh 2419580247


An
va an
Ket qua: USCLN = 345654321
e a con tro len dong bieu thc sa thanh 2419580247
An
va an
Man hnh hien 2.661538272 10 10
64

11

ay lai gap tnh trang tran man hnh. Muon ghi ay u so ung, ta
a con tro len dong bieu thc xoa ch so 2 e ch con
11 va an
419580247
Man hnh hien 4615382717
Ta oc ket qua
BSCNN = 26615382717
V du 3:
Tm cac c nguyen to cua

A = 1751 3 + 1957 3 + 2369 3


Giai
Ghi vao man hnh 1751
1957 va an
17
May hien
19
Chnh lai man hnh thanh 1751
17 va an
Ket qua: c so chung ln nhat cua 1751 va 1957 la 103(la so
nguyen to). Th lai 2369 cung co c so nguyen to la 103
3

Suy ra A = 103 (17


3

+ 19

+ 23 )

Tnh tiep 17 + 19 + 23 = 23939


Chia 23939 cho cac so nguyen to , ta c 23939 = 37 647
(647 la so nguyen to)
Ket qua: A co cac c nguyen to la 37 , 103 , 647.
5.

Quy ong mau so nhieu phan so


a. Cua 2 phan so:
Ta dung phng phap tm boi chung nho nhat e tm mau so
chung roi quy ong bnh thng
V du:
Tm mau chung nho nhat cua cac phan so sau:
3
7
va
a.
20
15
4
5
va
b.
21
18
Giai

a.
An 15

20

3

4

BSCNN (15,20) = 15 4 =60


Mau chung nho nhat la 60
65

b.
An 18

21

6

7

BSCNN (18,21) = 18 7 = 126


Mau chung nho nhat la 126
b. Cua nhieu phan so
Chung ta ap dung tm mau chung nho nhat vi 2 so roi lay ket
qua tm tiep vi so tiep theo, c vay cho en khi khong con mau
nao.
V du: Tm mau so chung cua cac phan so sau:
2
3
1
a.
,
,
5
25
4
5
3
2
7
,
,
,
b.
12
8
5
36
Giai

a.
An 5

25

1

5

BSCNN (5,25) = 5 5=25


Tiep tuc tm BSCNN cua ket qua vi mau con lai.
4
An 4
25

25
BSCNN (4,25) = 4 25 = 100
2
3 1
Vay mau chung nho nhat cua
,
,
la 100
5 25 4
b.
2
12
An 8

3
BSCNN (8,12) = 8 3 = 24
5
24
An 5

24
BSCNN (5,24) = 5 24 = 120
3
120
An 36

10
BSCNN (36,120) = 10 36 = 360
Vay mau chung nho nhat la 360

66

c. Viet phan so di dang 1 phan so co mau cho trc


V du: Viet cac phan so di ay di dang phan so co mau so 24
5
7
9
,
,
3
6
12
Giai
An
(Ratio)1(a:b = X:d)
(a)5
(b) 3
(d)24
(40)
5
40
=
Vay
3
24
Tng t tren, ta co:
7
28
=
6
24
9
18
=
12
24
Bai tap thc hanh
1.Tm mau chung nho nhat cua cac phan so sau:
7
8
a.
va
15
25
3
7
va
b.
12
15
5
3
2
7
,
,
,
c.
21
18
15
20
5
5
7
9
11
,
,
,
,
d.
4
6
8
10
12
ap so:
a. 75
b. 60
c. 1260
d. 120

2. Viet cac phan so di ay di dang phan so co mau so 48


5
7
9
7
6
,
,
,
,
3
6
12
24
36
ap so
5
80
7
56
9
36
7
14
6
8
=
,
=
,
=
,
=
,
=
3
48
6
48
12
48
24
48
36
48
6.

So sanh 2 phan so:


V du :
So sanh cac phan so sau:
14
41
a.
va
21
72
67

b.

38
va
135

251
344

Giai
a. An
(Ratio)1(a:b = X:d)
(a)14
(b) 21
(d)72
41
14
>
V 48 > 41 nen
72
21

b. An
(a) 38

(48)

(Ratio)1(a:b = X:d)
(b) 135

V 96.83 < 251 nen

(d)344

13072

135

(96.83)

38
251
<
135
344

Lu y :
+ Ngoai ra ta co the dung cach quy ong mau so nh tren a
hng dan e so sanh 2 phan so.
+ Ta con co 1 cach rat hay va nhanh la s dung phep tr 2 phan
so e so sanh
Bai tap thc hanh
1. So sanh cac phan so sau:
19
51
a.
va
23
73
43
129
va
b.
131
911
91
66
va
c.
325
275

ap so:
19
51
a.
>
23
73
43
129
>
b.
131
911
91
66
>
c.
325
275
7.

Phep nhan-Phep cong-Phep tr tren phan so:


a. Tnh toan c ban:
V du:
2 14 10 7
3 5


a.
3 21 29 12 7 4
68

34
14

123 35
7
54
31 1

15 345 78 6

b.
c.

a. An 2

14
3

12

Giai
21

10

29
1453

2346

Tng t, ta tnh c :
416
b.
615
53
c.
897
b. So sanh 2 phan so
V du: So sanh cac phan so sau:
8
9
a.
va
15
17

45

b.

29
2
9

c.

35
486
3
4

va
va

Giai
1

255
8
9
Ket qua khong co dau tr nen ket luan
>
15
17

a. An 8

b. An

15

5
3

486

17

Ket qua co dau tr.


Vay
c. An 2

45
29
9

35
<
486

Ket qua co dau tr. Vay

4
2
3
<
9
4
69

19

36

Bai tap thc hanh


Tnh
3 17 2 5 4 7
a.


7 24 3 8 9 6

b.
ap so:
a.
b.
8.

6 3 5 3 7 7


9 4 7 11 9 4
83
84
961
1386

Phep chia 2 phan so:


a. So nghch ao
V du:
1
a) Tnh
: an 8
Ket qua : 0,125
8
1
: an 8
16
Ket qua : 0,5
b) Tnh 8
16
5 7

: an 5
2
7
3
c) Tnh
2 3
29
an tiep
Ket qua : 4,8(3).
Ket qua:
6
Bai tap thc hanh
1
1 25
5 34
3
Tnh
, 186 ,
5
6 3
7 21
ap so: 0,2 ; 31 ;
8,(857142)

b. Phep chia phan so


V du: Tnh
3 9
a)
:
4 2
5 2
:
b)
6 3
Giai

a) An (Math)3

1

6

b) An (Math)5

5

4

70

9.

Hon so :

Lap hon so a

b
an
c

V du: Tnh
2
3
+2
a. 1
3
4
2
5
1
b. 2
3
6

)a

7
3
:1
8
4
4
3
d. 3
:2
9
5

c. 2

a. An 1
53

12

Giai
)2
3
5
4

12

) 3

Tng t:
b.

5
6

c. 1

9
14

d. 1

38
117

10. So thap phan:

Tnh 3, 375 + 7,425 - 4,5


Giai : An 3

375

425

Bai tap thc hanh


Tnh :
a) - 5,125 + 4,635 + 4,625 - 1,135
1
+ 6,5 - 2,436
b) 2,715 + 2
7
1
1
-3
+ 0,12
c) 10,75 +
4
7
ap so

a) 3
b) 8,9219
c)

1871
175
71

Ket qua :

63
10

11. T so :
T so cua 2 so:
V du: Tm t so cua cac so sau:
3
7
va 10
c.
va
a.
4
3
4
4
d.
va
b. 2 va
5
9

a. An 3

10

6
7
7
4

Giai
3

40

Tng t, ta co:
1
b. 2
2
13
c. 2
18
16
d.
63
12. Phan tram
V du 1:
a) Tnh 26% cua 86

An 86

26

559

25

(%)

Ket qua : 22,36.

b) Tnh 2,3526% cua 3000


An 3000

2.3526

(%)

Ket qua: 70,578.


c) Tnh 6% , 15% , 35% cua 3500
6
(%)
An 3500
15
(%)
3500
35
(%)
3500

35289

500

Ket qua : 210.


Ket qua : 525.
Ket qua : 1225.

V du 2:
Tnh t so phan tram cua cac cap so sau :
a) 45 phut va 2 gi b) 28 phut va 80 phut
c) 2454 m va 4 km
Giai
a) 45
120
(%)
Ket qua : 37.5%
b) 28
80
(%)
Ket qua : 35%
c) 2454
4000
(%)
Ket qua : 61.35%
72

V du 3: Bai toan ve t le xch


Tnh ng dai thc te cua 2 iem cach nhau 3,5 cm tren ban o t
le 1/50000
Giai
Ghi vao man hnh
3.5
5
4
(175000)
hay
3.5
5
10
4
(175000)
Ket qua 175000 = 1,75km
Bai tap thc hanh
1) Tnh 9% , 18% , 38 % , 65 % cua 1250
S: 112,5;
225;
475;
812,5
2) So cay Lan , Hang , Phng moi ngay ti c lan lt la 28,
30, 40 cay . Hoi so cay moi ngi ti c trong mot ngay neu :
a) Nang suat lao ong cua Lan tang 25 %
b) Nang suat lao ong cua Hang tang 10 %
c) Nang suat lao ong cua Phng giam 35 %
b) 33,
c) 26
S: a) 35,
IV.

So o Goc Cac phep tnh


Tnh toan khi man hnh hien D: an
3 (Deg)
Dung phm
e ghi o, phut, giay va phm
chuyen phan le thap phan ra phut, giay.

(hay

V du 1: oi 455739 ra so thap phan va ngc lai


Giai
Chnh tren man hnh che o D bang cach an phm
An 45
57
39
e ghi vao man hnh
455739 va an
may hien 45.96083333 (oc 45.96083333)
may hien lai 455739
an tiep
V du 2: Tnh
a) 455739 + 345658 - 254251
b) 455739 7
c) 1345658 4
d) 1345658 254251
Giai
Ghi vao man hnh
a) 45
57
39
34
56
51 va an
.
Ket qua : 5511 26
73

58

) e

3(Deg)

25

42

Giai tng t cho cac bai sau.


b) 32143 33
c) 3344 14,5
d) 5,248058247
V du 3: Bai toan ve gi, phut, giay (cung tnh tng t nh o, phut,
giay)

a) Tnh 2 g 47 ph 53 gi + 4 g 36 ph 45 gi
Giai

Ghi vao man hnh


2
47
53

36

45

va an

May hien:7 o 2438. oc 7 g 24 ph 38 gi


b) Tnh thi gian e mot ngi i het quang ng 100 km bang van
toc 17,5 km/g.
Giai

Ghi vao man hnh


100
17.5 va an
Ket qua

5 o 42'51,43"

c) Tnh ng dai d i c trong 5 g 42 ph 51 gi


17,5km/h

vi van toc

Giai

Ghi vao man hnh


Ket qua

17.5 5 o 42'51" va an
d 100 km

d) Tnh van toc di chuyen cua mot ngi biet trong 5 g 42 ph 51 gi a


i het quang ng 100 km
Giai
o

Ghi vao man hnh 100 5 42'51


va an
Ket qua v 17,5km/g
e) oi thanh o, phut:
V du: 17,25 o =17 o 15 = 17

An 17.25

1
4

69

(17o150)

74

Bai thc hanh


1) Tnh ra gi , phut , giay cac cau sau
a) 2 gi 45 phut 30 giay + (3gi 15phut 0 giay) 3.
(ap so: 12 gi 30 phut 30 giay)
1
(4gi 40 phut 40 giay) + 2,5 gi
b)
4
(ap so: 3 gi 40 phut 10 giay)
1
c) 40 phut 50 giay + (6gi 36 phut 18 giay)
6
(ap so : 1 gi 46 phut 53 giay)
d) 150 phut 45 giay + 1,5 gi + 3600 giay
(ap so : 5 gi 0 phut 45 giay)
2) Tnh thi gian oto i het quang ng 450km vi van toc
48 km/gi .
(ap so : 9gi 22 phut 30 giay)
3) Trong 3 gi 30 phut 45 giay oto i het quang ng 160 km. Tnh
van toc oto.
(ap so : 45, 55 km/gi)
4) Tnh quang ng oto i c trong 4 gi 15 phut 30 giay vi van
toc 48 km/gi.
(S : 204,4 km)
LP 7
I. So hu t- so thc
Neu mi va chnh may an
3 (ALL) th may s dung dau
cham () lam dau cach gia phan nguyen va phan le thap phan
V du 1: Tnh
2 5
8
5
6
5
a) (6
)(

+4
) + 4 (3 +
)
3 7
11
3
7 13

7
15
3 8

)
b) (5
9
13 11
7 3

c) 7,2 [6,25 - (- 3,42) + 7,54] 9,83


d) (3) 2
e) 5 4
5
f)
7

3
g)
4

75

h) 2.41 3
i) (5.2) 4

Giai
Ghi vao man hnh y het nh e va an
sau moi bieu thc.Ta
c ket qua
16894
878
( ay, may khong oi ra 16
khi an
a) 16.8771... =
1001
1001

c v phai dung hn 10 k t)
b)

898
39

Ghi chu: che o

Khi ghi vao man hnh 4


3
2 hay 4
4
8
3
2 may eu hieu la
2=
do dau
u tien hn
3
3
phep nhan tat. Cung nh ghi
4
4 3 (trong che o Line) may van hieu
= 4.1887, con muon
3
4
th phai ghi 4
3
()
(hay viet tat la
ghi
3
4

( ))va an

e co ket qua 0,4244

c) 12.6055(khi ghi vao man hnh , dau ngoac [ c thay bang dau
( (v may khong co dau ngoac vuong)
d) 9
e) 625
3125
(khong c ghi vao man hnh 5
7
5
f)
16807
5
v may se hieu la 5 do phep luy tha u tien hn
)
7
81
g)
256
h) 13,997521
i) 731,1616 (so am phai c at trong dau ngoac n)
V du 2: Tnh

a) 7 3

b) 10 6

c) 5 10 3 10 6
a)

An 7

Giai

3
76

b)

c)

Ket qua 0.002915 = 2.915 10 3


An
(
)
6
1
= 0.000001 = 10 6
Ket qua
1000000
An 5

Ket qua

5 10 9

V du 3: ien dau thch hp vao o trong


22
a) 0.5
40

An
22
40
Ket qua : - 0,55 ien dau >
25
78
28
8
25
9
Lam tng t nh tren, ta ien dau >, >.

b)

V du 4:
a) Tm x , biet

x2 +

3
2
=2
7
9

Giai

Dung may tnh 2

85
63

85
85

2
x
2
x

85
63
63
x2 =


85
85
63

x 2 63
x 63 2
211
63
2
An 85
63

b)

85

63

x
Vay

211
63
41
63

41
63

6 1
7

=
+ 0,9
5 x
5

77

Giai
7
6 1
=
+ 0,9

5
5 x

6 1
5 x

6 1
5 x

An

10
11

1
6 7

7
0,9
0,9
x
5
5
5

1 6 7
7

0,9
0,9
5

x 5 5
5

0.9

10
11
An
e a con tro len dong bieu thc va sa lai thanh
6
7
( + ( + 0.9))1
5
5
2
Ket qua :
7
Ket qua :

Bai tap thc hanh


1) Tnh gia tr bieu thc:
1 7 3
5
2
a)
4 15 7
3

S:

2021
420

3 2 1 2
21
1187
5
b)
S:
2 :
7
13
336
8 5 3 7

2) ien dau thch hp vao o trong


21
17
43
a)
5
4
10
22
335
b)

7
113
c)

21
5
22
S:
7

S:

15625

62 52 10

43

106

26
S:

1
43

15625
106

17
43

4
10
335

113

3) Tm x, biet
1
a) x 3 =
2

2
+
3

x
S:
x

78

35
27
197
27

62 52 10
26

b) 3

c)
II.

2 1
3
=

5
7 x

1
+ 2
3

1575

x 3967
S:
1575

x 14317
x 1
S:
x 5

1 1
9
3
=

2x 5
5
2

Luy tha cua mot so hu t:


V du 1:
1
Tnh
3

an

3
2 4 an

1 2

4

3
(

)3

an

V du 2:
Tm x, biet
1
a) x :
3

An 1

Giai
1

7
7
b) .x =
2
2
7
x=
2

.
1
65536

1
9

4
Ket qua :

7
:
2

(2,5)2. An 2

1
729

6
Ket qua:

4
49

Ket qua :

25
4

V du 3:

Tnh

1331
64

. Ket qua:

Ket qua :

An 7

1
9

Ket qua:

5
79

(1,25)3. An 1

25

Bai tap thc hanh


1
1. Tnh :
4

2
;
3

Ket qua :

7
;
3

2 1
;
3 7

S:
2. Tnh (0,6)2(0,4)3

1
a) (2,5) (1,5)
4
2

+ (3,8)2

b) (2,4)2 + 1,602 7,326 (3,2)3


1
1
8,5 (2,5)3
6
c) 4
1
(2,7)3 .(0,5)3 .5,4
8
4. Tm so n N sao cho
1.02n n

n1
n1
1.02
Dung may ta tnh

1.02 10

; 1

1
8
243
289
;
;
;
; 25
64 16 16807
441

1
102 82 103.
36

3. Tnh

125
64

S: 0

S: 20,6768
S: -15,2717
S: 0,2413

= 1.22

1.02

10

1.02

200

300

= 7.24
= 52.48

= 380.23
1.02
Ta thay
200 < n < 300
Tiep tuc th nh the , ta c
1.02 285

1.02 286
III.

= 282.52
= 288.17

Ket qua n = 285

T le thc
V du 1: Kiem xem cac so sau co lap thanh t le thc khong:
a) 3 :19 va 5,13 : 32,49
b) 63:117 va 12,51 : 23,27
Giai
Cach 1: Chung ta co the nhap trc tiep an
may se t n gian
va xem ket qua cuoi 2 phan so co bang nhau khong.
Cach 2: Dung chc nang Ratio cua may
80

a) An:

(Ratio)

513
x
100

(a) 3

(b)19

(d) 32,49

(b) 117

(d) 23,27

5,13

Vay co the lap c t le thc


b) An:
(Ratio)
(a) 63
1253
12,53
x
100

V 12,53 12,51 nen ta khong the lap c t le thc


V du 2: Tm x trong cac t le thc sau:
3
1
a) 7,6:(2x) =
:3
7
5
b) 0,02:2,5 = (0,8x):0.8
2
4
c) 1
: 0,9 =
: (0,3x)
3
3
Giai

a) 7,6:(2x) =

3
1
3
1
3
1
:3
(7,6:2):x =
:3

:3
= (7,6:2):x
7
5
7
5
7
5

(a)3

An
2128
75

(b)3

(c)7,6

(28,37(3))

Vay x = 28,37(3)
b) 0,02:2,5 = (0,8x):0.8 0,02:2,5 = x:(0,8 :0,8)
An
1
125

(a) 0,02

(b) 2,5

(d)0,8

0,8

(8 10 3 )

Vay x = 8 10 3
2
4
2
4

: 0,9 =
: (0,3x) 1
: 0,9 = : 0,3 : x
c) 1
3

3
3
3
An
0,3

(a)1
12
5

(b) 0,9

(2,4)

Vay x = 2,4
IV.

Tnh chat cua day t so bang nhau


x
y
V du 1: Tm 2 so x va y, biet
=
va x + y = 11
3
6

81

(c)4

Ta co:

Giai
x
y
xy
11
=
=
=
3
6
36
9

(a) 11

Nhap vao may:


Vay x =

(b) 9

22
.
3

(d) 6

22
3

V du 2:

Tm x, y, va z biet

x
y
z
=
=
6
2
4

va

11
.
3

(d) 3

11
3

Dung dau sa lai thanh (a)11


Vay y =

(b) 9

x + 2y + 3z = 23

Giai
x
y
z
x
2y
3z
x 2y 3z
23
=
Ta co :
=
=

=
=
=
6
2
4
6
4
12
22
22

Nhap vao may :


Vay x =

69
11

(d)6

(a) 23

(b) 22

(d) 4

46
.
11

(a)23

(b) 22

(d)12

138
.
11

23
11

An :
Vay z =

(b) 22

69
11

An:
Vay y =

(a)23

46
11

V du 3: Co 24 t giay bac loai 2000 , 5000, 10000. Tr gia moi


loai tien tren bang nhau.Hoi moi loai co may t ?
Giai
Goi x, y, z lan lt la so t bac 2000, 5000 , 10000.Ta co he pt
sau:
x y z 24
x y z 24

x y

2x 5y 10z
5 2 z

Lam tng t tren, ta c z = 3, y = 6, x = 15


82

V.
1.

So thap phan hu han.So thap phan vo han tuan hoan


Viet mot phan so di dang so thap phan vo han tuan hoan:
V du: Viet cac phan so sau di dang so thap phan va di dang
thap phan vo han (neu c):
5
7
7
9
,
,
,
9 18
45 13
Giai

Nhap phan so bnh thng: 5


Roi an
5
Ket qua:
= 0,(5)
9
Tng t, ta co ket qua sau:
7
= 0,3(8)
18
7
= 0,1(5)
45
9
= 0,(592307).
13
2.

Viet mot so thap phan vo han tuan hoan di dang phan so:
Cach 1: S dung chc nang chuyen oi co san cua may
V du: Viet cac so sau di dang phan so
0,(13) ; 1,1(12) ; 0,3(8) ; 0,5(2)
13
An 0
(
)13
99

Ket qua ra:

0,(13) =

An 1

13
99

)12

367
330

Tng t, ta co cac ket qua sau:


7
0.3(8) =
18
47
0.5(2) =
90
Han che cua cach nay la ket qua la 1 phan so toi gian (ke ca dau
ky hieu phan so) th may se khong bieu dien di dang phan
so.Cach sau tong quat hn va giai c vi moi so
Cach 2:
V du 1: Phan so nao sinh ra so thap phan sau:
83

a) 0,12
b) 1,345
c) 0,123123123. . . . . (ghi tat
d) 4,353535. . . . . . . . (ghi tat
e) 2,45736736. . . . . . (ghi tat

0,(123))
4,(35))
2,45(736)).

Giai

a) 12 / 100
b) 1345 / 1000
c) 123 / 999
d) 4 + 35 / 99 = 431 / 99 = (435- 4) / 99
e) 2 + 45 / 100 + 736 / 99900 = 245491 / 99900
= (245736 245) / 99900
(Mau so la cac so 9 va cac so 0 tiep theo, co bao nhieu so 9 la do
cum tuan hoan co bay nhieu ch so , co bao nhieu so 0 tiep theo la
do cum tuan hoan au tien cach dau phay bay nhieu ch so .
T so bang so a cho vi cum tuan hoan au tien khong ghi dau
phay tr cho phan khong tuan hoan khong ghi dau phay (tham khao
k v du e))
Ket qua bai e) khong oi ra hon so c v phai dung hn 10 k t.
V du 2: Tm ch so le thap phan th 105 cua phep chia 17/13
Giai
Thc hien phep chia 17 13 = 1.(307692) (thc ra la
1.307692307692...) Ta thay chu k la 6 , mat khac 105 3 (mod 6).
Suy ra ch so le thap phan th 105 cua phep chia 17/13 la 7.
V du 3: Tm so n N nho nhat co 3 ch so biet n 121 co 5 ch so
au eu la so 3.
Giai

Ta khong the dung may e tnh n 121 vi n co 3 ch so, nhng ta


biet 123 121 , 12.3 121 , 1.23 121 co cac ch so giong nhau

Do o ta tnh 1.00 121 = 1 ; 1.01 121 = 3.3333. . . .


Ket qua n = 101
3.

Cac phep tnh tren so thap phan vo han tuan hoan:


V du: Tnh
a) 1,2(34) + 2,(15)
c) 4,2(51) 12,(7)
b) 15,(15) 16,1(16)
d) 16,(22) 4,(11)
Giai
Nhap bieu thc tng cau giong nh e vi
(
e ch phan chu k tuan hoan.
84

) la phm

S:
a) 3,3(85)
b) 0,9(64)

c) 54,3(249158)
d) 3,(945)

Bai thc hanh


1. Viet cac phan so sau di dang so thap phan va di dang thap
phan vo han tuan hoan (neu c) :
44
a.
S: 0,(483516)
91
59
b.
S: 0,(6781609195402298850574712643)
87
35
S: 0,28
c.
125
19
S: 0,8(63)
d.
22
2. Viet cac so thap phan vo han tuan hoan sau di dang phan so
331
a. 0,8(945)
S:
370
82263
b. 0,82(345)
S:
99900
4481
c. 0,13(456)
S:
33300
VI.

Lam tron so
May co hai cach lam tron so
+ Lam tron so e oc (may van lu trong bo nh en 15 ch so
e tnh toan cho cac bai tiep sau) NORM hay FIX n
+ Lam tron va gi luon so a lam tron cho cac bai tnh sau FIX
n va Rnd
V du 1: An 17
13
sau o nhan
may hien ket qua la
1,307692308 nhng trong bo nh th ket qua la 1,30769230769231
(may gi u 15 ch so va ch 15 ch so)
cho en khi hien gia tr so thap
Neu chon FIX 4 sau o an phm
phan th may hen ket qua la 1.3077 nhng trong bo nh th ket qua
la 1.30769230769231 (may gi u 15 ch so) v the khi an tiep
13
ta c ket qua la 17.0000
V du 2: An 17
13
may hien ket qua la 1.307692308 nhng
trong bo nh th ket qua la 1,30769230769231 (may gi u 15
ch so)
Neu chon FIX 4 va an tiep
(Rnd)
th may hien ket qua
85

la 1.3077 va gi ket qua nay trong bo nh(ch co 4 ch so le va a


lam tron) v the khi an tiep
13
ta c ket qua la
17.0001
VII. So vo t - Can bac 2
V du 1: Tnh
a)
289

b)

15129

c)

5,4756

d)

225
361

e)

f)

1234

g)

35.17

h)

123
789

k)

45

l)

452 73

m)

789 38

n)

o)

74 + 5 8

59
48
3
19

62
14

Giai
Ghi vao man hnh y het nh e va an
Dau phan so ghi bang phm
Ket qua
a)
289
= 17

b)

15129

c)

5,4756 = 2,34

d)

225
361

e)

= 2,6458

f)

1234

= 35,1283

g)

35.17

= 5,9304

sau moi bieu thc

= 123
15
19

(ghi

86

225

361va an

h)

123
789

k)

45

l)

452 73 = 22,9129

m)

789 38 = 173,1531 (ghi

n)

o)

74 + 5 8

= 0,3948 (ghi

123

789 va an

= 5 2
(ghi

59
= 2,8686 (ghi
48
3
19

452

73 va an

789 38 va an

59

5
An
Ket qua: 1

48 va an

62
= 17,7732
14

49

10

11

Bai tap thc hanh :


Tnh gia tr cua bieu thc
7
4
A= 1 + 2
9
16

B = 2 121 92

132 144
0,0001

S:

22 (32 22 )2
153 143
2,25

S: 79,(3)

V du 3:
Tm x , biet
a) 169x2 = 121
121
121
11
x2 =
x=

169
169
13

An
An
b)

121

169
121

169

4x2
5.20
= 5 x2 =
= 25
20
4
25
An
25
An
Ket qua: x = 5
87

17
6

S: 2 10

10

)
)

V du 2 : Tnh gia tr cua bieu thc


5 49
A=
102 11 22

C=

c)

x = 0,5
x = (0.5)2 .

An 0.5

Ket qua: x = 0,25


d) 2x 1 = 6,4
2x 1 = 40,96.
x = 20,98

An

3
V du 4: Muon tnh

2
3
Ket qua:

6.4

th phai ghi vao man hnh


2

va an

3
4

Bai tap thc hanh :

a)

x2
=
3969

25 102

S: 140,8723
19
hay 2,(714285)
7

b) (x2 1) 49 = 312

S:

c) 3 x = 50,43

S: 282,5761

3x2 2 =

d)

S:

29

VIII. ai lng t le
V du 1: Cho biet x va y t le thuan . Hay ien so thch hp vao bang
sau :

x
y

4
12

6,3

Giai

Tm he so
(Ratio)
(a:b=c:X) 4
12
5
(15)
An
An
va sa c lai thanh 6 roi tnh tiep c ket qua la 18
va sa c lai thanh 6.3 roi tnh tiep c ket qua la 18,9
An
Ta c bang sau:
x
y

4
12

5
15

6
18

6.3
18.9

V du 2: Cho biet x va y t le thuan . Hay ien so thch hp vao bang


sau
88

x
y
An
An
An

-4

7
- 28

40

49.2

(Ratio)
(a:b=c:X) 7
-28
3
(-12)
va sa c lai thanh -4 roi tnh tiep c ket qua la 16
va sa c lai thanh 5 roi tnh tiep c ket qua la -20

An
(Ratio)
(a:b=X:d) 7
-28
40
(-10)
An
va sa c lai thanh 49.2 roi tnh tiep c ket qua la -12,3
Ta c bang sau:
x
y

3
-12

-4
16

5
-20

7
- 28

-10
40

-12,3
49,2

V du 3: Dien tch hnh ch nhat bang 1600m2. Tnh o dai moi


canh, biet chieu dai va chieu rong t le vi 25 va 16.
Giai
Goi a , b la hai canh cua hnh ch nhat (a > b > 0)
S ab 1600

Ta co a
b
25
25 2
2 1600 16
25 16 a 16 b 16 b 1600 b
25

An
1600
Ket qua: b = 32 m
Tnh a: An 1600
Ket qua: a = 50 m

16

V du 4: Tm x , y, z biet
4
7
12
=
=
x
z
y

25

va

2x + 3y + 4z = 1925

Giai
4
7
12
8
21
48
8 21 48
=
=
=
=
=
=
x
z
2x
4z
y
3y
2x 3y 4z

An

(Ratio)
(a:b=c:X) 8
21
48
1925
4
(100)
Bai tap thc hanh
1. Cho x, y la hai ai lng t le thuan . ien so thch hp vao o
trong trong bang sau
x
y

-15

-13

-12
48

7
-60

89

-84

-108

S:
X
Y

-15
60

-13
52

-12
48

5
-20

7
-28

15
-60

21
-84

27
-108

2. Chu vi hnh ch nhat la 2100m .Tnh o dai moi canh, biet t le


chieu dai va chieu rong lan lt la 20 va 15.
S : a = 600 m ; b = 450m
y
x
=
va 2x + y = 1782
3. Tm x , y biet
32
17
S : x = 459 ; y = 864
x
y
z
4. Tm x , y ,z , biet
=
=
va 6x + 8y + 9z = 4161
5
9
13
S : x = 95 ; y = 171 ; z = 247
C lan lt t le vi 6,12,
ABC so o cac goc A, B,
5. Trong
36.Tnh so o moi goc .
40o , C 120o
S : A 20o , B
IX.

T le nghch
V du 1: Cho biet x va y t le nghch . Hay ien so thch hp vao
bang sau

x
y
An
An
An

-12

-7

-4

5
630

450

Giai
-12
5
630
va sa -12 thanh -7 roi an
va sa -4 thanh -7 roi an

-12
-262,5

-7
-450

-4
-787,5

150

(-262,5)
c ket qua -450
c ket qua -787,5

An
630
450
5
va sa 450 thanh 350 roi an
An
An
va sa 350 thanh 150 roi an
Ta c bang ket qua nh sau:
x
y

350

5
630

(7)
c ket qua 9
c ket qua 21
7
450

9
350

21
150

Lu y: oi vi ham t le nghch lap t le thc theo hang ngang va


chieu b ao lai, con t le thuan ap dung theo hang doc.
V du 2: Ba oi may bm nc gom co 31 may (gia thiet cac may
eu cung cong suat), bm nc vao ba cai be co cung the tch. oi
th nhat hoan thanh cong viec trong 4 ngay, oi th hai hoan thanh
90

cong viec trong 6 ngay, oi th ba hoan thanh cong viec trong 10


ngay. Hoi moi oi co bao nhieu may ?
Giai
Goi so may bm cua moi oi lan lt la x , y , z (x , y , z )
Ta co : x + y + z = 31. Do so may bm t le nghch vi so ngay lam
viec nen : 4 x = 6 y = 10 z
x
y
z
xyz

Hay
1
1
1
1 1
1

4
6 10
4 6 10
31
=
1 1
1

4 6 10
Tm x:
(a)31
(b)
1
4+1
6+1
10
4
(d)1
Ket qua: x=15
Tm y:
An
dung
sa gia tr d lai la 1
6 roi an
Ket qua: y= 10
Tm z:
An
dung
sa gia tr d lai la 1
10 roi an
Ket qua: z= 6
Bai tap thc hanh
1) ien so thch hp vao o trong, biet x, y la hai ai lng t le
nghch
x
y

-15

-12

9
1980

990

810

712.8

ap so:
x
y

-15
-1188

-12
-1485

6
2970

9
1980

18
990

22
810

25
712.8

2) Bon oi xe ch hang gom 61 chiec (gia thiet cung tai trong). ch


cung mot khoi lng hang t iem A en iem B. oi xe th nhat
hoan thanh cong viec trong 2 ngay, oi xe th hai trong 4 ngay,
oi xe th ba trong 6 ngay, oi xe th t trong 10 ngay.Tnh so xe
cua moi oi.
S: 30 , 15 , 10 , 6

91

X.

Ham so
V du 1: ien cac gia tr cua ham so y =

4
vao bang sau
x

Giai

Ghi vao man hnh:


4

4.5 va an

8
9

Ket qua y =

An

e a con tro len man hnh chnh lai thanh


4
3 va an
.
Ket qua y =
4
3
Lam tng t nh tren, ta c bang ket qua
2
3

4,5

8
9

4
3

2,4
5
3

2
5
20
17

4 3
0,5774

V du 2: ien cac gia tr cua ham so y = 3x vao bang sau

5,3

4
3

2,17

3
7

5 7

y
Lam tng t nh tren, ta c bang ket qua
x

5,3

4
3

2,17

15,9

12

6,51

3
7
2
13
7
4

5 7
39,6863

V du 3:

Tnh gia tr cua ham so y - f(x) = 4x2 + 5 tai x = 1, x = 3, x =

1
4

Giai
An 1
(STO)
(A) (Gan 1 cho A , dung A thay cho x)
An tiep
(:)4
(A)
5
Ket qua: f(1) = 9
An
sa lai la : 3 A : 4A2 + 5 an
Ket qua:
An tiep

f(3) = 41
a con tro ve au dong bieu thc, an
(ins)e ghi chen vao man hnh

1
A : 4A2 + 5 an
4

92

e xoa 3 , an

1
21
1
f = 5
=
4
4
4

Ket qua:

Bai tap thc hanh


2
1) Cho ham so y = f(x) = 3x2 5x + 4.Tnh f(2) ; f(-4) ; f
3

S: 6 ; 72 ; 2

2x 1
. Tnh f(0) ; f(-2) ; f(4)
x3
ieu g se xay ra neu ban tnh f(3) ?

2) Cho ham so y = f(x) =

S:

1
3

3
5

; 9 ; vi f(3) may se bao loi Math Error .V f(x)

khong xac nh tai x = 3 .


3) Cho ham so y = f(x) =

1
2x 1 .Tnh f 1 , f(4) , f(40)
2

S: 2 ; 3 ; 9
XI.

Thu thap so lieu thong ke


Goi chng trnh thong ke
An
2 (STAT)
Xoa bai thong ke
2(Memory)
(Yes)
V du: iem cac mon hoc cua mot hoc sinh lp 7 c cho bang
sau :

Mon

Toan

Van

Sinh

iem

Cong
nghe
8,5

Am
nhac
6,5

1) Hay nhap d lieu t bang tren vao may tnh


2) Chnh sa d lieu bang cach
- Sa iem L thanh 7,5
- Xoa iem 4 cua mon Sinh hoc
- Them iem mon Giao duc cong dan la 8
Giai

2 (STAT)
Chnh hien cot Freq
7
9
6
5
4
1) 8
2) Sa iem L thanh 7,5
([ STAT]) 2 chnh
Xoa iem 4 cua mon Sinh
93

8,5

6,5

ti gia tr th 5 roi nhap 7,5

Di chuyen xuong dong 6 co gia tr la 4 bam


e xoa
Them iem mon Giao duc cong dan la 8
Di chuyen xuong dong khong co gia tr roi nhap 8
Xoa d lieu thong ke va nhap
Thoat khoi chng trnh :
Bai tap thc hanh
Cho bang sau

STT
Gia
tr

1,25

2,4

3,7

-5

6,12

1
7

7
4
1
9

0,1

Hay:
a) Nhap d lieu t bang vao may tnh
b) Chnh sa d lieu bang cach :
2
- Them gia tr 3
vao bang d lieu
7
- Xoa gia tr - 5 va 0,1
- Sa 2,4 thanh 5
- Thoat khoi chng trnh thong ke
XII. Bang gia tr tan so:
V du 1: iem hoc k 1 cac mon hoc cua mot hoc sinh c cho
theo bang sau :
Hay :
a) Nhap d lieu t bang vao may tnh
b) Chnh sa d lieu bang cach :
- Xoa bt (5 ;2) va Tnh X
- Them gia tr (1 ; 2) vao bang d lieu .Tnh tan so
- Sa (7, 5 ; 5) thanh (8, 5 ; 6). Tnh tan so va X
Giai
iem
7,5
8
9
10
6,5
5
4
Tan so
5
4
7
5
3
2
3

An
2(Memory)
e xoa thong ke cu
2 (STAT)
Vao chng trnh thong ke
Hien cot Frequency:
(STAT)
a) Nhap d lieu t bang a cho
2(Data)
An
Sau o nhap vao bang nh sau

94

2
1

(ON)

x
7,5
8
9
10
6,5
5
4
2

Freq
5
4
7
5
3
2
3
1

b) Xoa (5,2) dung phm

di chuyen ti dong co gia tr (5,2)

roi an
e xoa. Tnh X an
(Var) 2
Ket qua: 7,71428
c) An
2(Data)
di chuyen xuong dong cha co d lieu va
Dung phm
nhap vao d lieu mi la (1,2)
(STAT)
(Var)
(n)
Tnh tan so : an
Ket qua: n = 30
d) An
2(Data)
Dung phm
di chuyen xuong dong co d lieu (7,5;5).
Ngay o 7,5 nhap 8,5 roi dung cac phm mui ten di chuyen sang
ben phai (ngang vi gia tr va sa) sa 5 lai thanh 6.
Tnh X an
Ket qua: 7,4677419
Tnh tan so : an
Ket qua: n=31

(Var) 2
(STAT)

(Var)

(n)

V du 2: Mot xa thu thi ban sung. Ket qua so lan ban va iem so
c ghi nh sau

iem
Lan ban

4
8

5
14

6
3

7
12

8
9

Tnh:
a) Tong so lan ban
b) Tong so iem
c) So iem trung bnh cho moi lan ban
Giai
Xoa bai thong ke cu
An
2(Memory)
(Yes)
2 (STAT)
Vao che o thong ke :
Hien cot Frequency:
(STAT)
95

9
13

(ON)

An
(STAT) 2 (Data).Sau o,nhap d lieu t bang a cho
Sau khi nhap xong, man hnh se co dang sau:
x
4
5
6
7
8
9

Freq
8
14
3
12
9
13

a) Tong so lan ban la n = 59: An


(STAT)
b) Tm tong so iem
Ket qua: Tong so iem la 393
(STAT)
c) Tm so trung bnh: An
Ket qua: iem trung bnh la 6,66
(Muon tm lai tong so lan ban th an

(Sum)
(var)
(STAT)

(59)
(x )
(x)
(var)

(n)

Ghi chu: Muon tnh them o lech tieu chuan va phng sai, ta thc
hien nh sau :
Sau khi a nhap xong d lieu, an
(STAT)
(var)
(X n )
Ket qua:

X n =1,7718

An tiep
Ket qua:

Phng sai X n = 3,1393

XIII. Bai toan ve n thc, a thc:


V du 1:
So 3 co phai la nghiem cua a thc sau khong?
3x4 5x3 + 7x2 8x 465 = 0
Giai
An
3
(STO)
(A)
Ghi vao man hnh
(A)
4
5
(A)
7
3
8
(A)
465
Va an
man hnh hien. Ket qua: 0
Vay 3 ung la nghiem cua a thc tren.
V du 2: Tnh gia tr cua y = 5x2 3x + 4 tai x = -2, x = 3

An

2
3

(x)

(STO)
4

Giai
(x)
96

(:) 5

(A)

(x)

Ket qua: 30
Vi x = 3 an tiep
e a con tro ve au dong, an
dau
, an 3 ghi e len , ta co man hnh:
.
3 X : 5X2 3X + 4, an
Ket qua:

40

V du 3: Tnh gia tr cua 3xy2 + 2x2y3 tai x =

An 1

(STO)

4
An tiep 3
(Y)
Ket qua: -8
V du 4: I =

An 2.41
-3.17
4
3

(STO)
(x)

6xy xz

1
, y = 4.
2

1
cho X)
2
(Y) (Gan- 4 cho Y)
(Y)
2

(X) (Gan

3x2y 2xz3 5xyz


2

e xoa

(x)

Vi x = 2,41; y = - 3,17; z =

4
3

Giai
(STO)
(X)
(STO)
(Y)
(STO)
(A)

Sau o ghi vao man hnh :


(3X2Y 2XA3 + 5XYA) (6XY2 + XA)
va an
Ket qua: I = 0,7918
Bai tap thc hanh
1) Tnh gia tr cua a = 2x3 4x2 + x 5 tai x = 1, x = 5
S: 12 ; 150
1
va y = 3; x = 4 va
2) Tnh gia tr cua B = 4xy2 + 3x2y y3 tai x =
2
y=2
27
; 152
S:
4
1
3) Tnh gia tr cua C = 4xyz + xy2z3 2xz tai x =
, y = 2, z = 3
2
S: 39

4) Tnh D =
S:

x2yz
xy y2z

tai x = 1 , y = 2 , z = 4

4
9
97

XIV. Hnh hoc


1. Goc oi nh va so le trong:
V du 1:
x

x
3

O
1

Cho O 2 = 60 o . Hay tnh so o cac goc con lai.


Giai

Ta co:

O 2 + O 3 = 180 o (V O 2 va O 3 ke bu)
O 3 = 180 o 60 o

(Deg)

An
An tiep 180

. Ket qua: 120 o

60

Vay O 3 = 120 o

Tnh O1 : V O1 va O 3 la 2 goc oi nh nen ta co: O1 = O 3 = 120 o .


Tng t: O 2 va O 4 la 2 goc oi nh, suy ra: O 2 = O 4 = 60 o .
V du 2: Cho x // y, O1 = 55 o , tam giac BOD can tai O.

1
y

C
1 2

A
1

1
2

O
4

1
3 2
B

3
D

Hay tnh cac goc con lai tren hnh.

98

Giai

Ta co:

O1 = O 2 (oi nh)
180o 55o

D4 = C4 = B4 = A4 =
= 62 o 30
2
2
= A2 = D2 = C2 = B

(Do tam giac BOD can va tnh chat so le trong)


Dung may tnh: an
Ket qua:

180

55

62 o 30

Ta co :
1 B
3 = (180 o 62 o 30) = 117 o 30
D1 A1 D3 A3 C1 C3 B

Dung may tnh : an


Ket qua:

180

62

30

117 o 30

Bai tap thc hanh

1) Cho A = 110 o , tam giac OAB can tai A, tam giac COB can tai O,
COA = 125 o , OK la phan giac goc COB .Tnh cac goc con lai.

1 = O 2 = 35 o , COB = 90 o , O 3 = COK = 45 o ,
S: B
O1 = 55 o , K1 = K2 = 90 o

2) Cho x z, y z, tam giac OAB vuong can tai O.Tnh so o cac


goc tren hnh

99

z
1

4A
2

6
5
B4

= 90O ;
1 = B
S: A1 = A3 = 45O ; A2 = A4 = 135O ; B
4
= B
= B
= B
= 45O
B
2
3
5
6

2.

nh ly Pitago
V du 1: Cho tam giac vuong ABC co hai canh goc vuong
AB = 12cm ; AC = 5cm. Tnh canh huyen BC ?
Giai
2
2
2
AB + AC = BC

BC =

122 52 = 13cm

An
Ket qua:

12
13cm

an

V du 2: Cho tam giac ABC co AH BC, AB = 5 , BH= 3, BC = 10.


Hay tnh AH , AC
A

H
Giai

Theo nh l Pitago , ta co
Trong tam giac ABH :

AB 2 = AH 2 + BH 2
AH 2 = AB 2 BH 2
100

AH =

52 32

Dung may tnh: An


5
3
An
Ket qua : AH = 4
Suy ra: HC = BC - BH = 7
Ap dung Pitago trong tam giac AHC, ta co:

AC 2 = AH 2 + HC 2 = 4 2 + 7 2 = 65
An
4
7
Ket qua: AC = 65 = 8,0623
Bai tap thc hanh

Cho cac tam giac vuong ABM, DMN, CNB nh hnh ve, co AB = BC
= AD = CD = 8, AM = 5, DN = 4 .Tnh chu vi tam giac BMN (Danh
cho HS lp 7 cha hoc hnh vuong)
S: 23,3783
3.

Quan he gia goc va cach oi dien trong mot tam giac:


V du:
Cho tam giac ABC co :
= 46 o 25
a) C = 70 o 16 , B

b) A = 60,5 o , C = 51,5 o
Hay so sanh o dai cac canh cua tam giac ABC trong hai trng
hp tren
Giai
o
+ C)
a) Tnh goc A : A = 180 ( B
An 180
Ket qua:

46

25

70

16

A = 63 o 19
. Vay AB > BC > AC
C > A > B

b) Tng t, ta co:
= 68 o C < A < B
. Vay AB < BC < AC
B
101

Bai tap thc hanh


So sanh cac canh cua tam giac CDE trong cac trng hp sau
= 49 o
a) C = 75 o , E
= 64 o 50
b) D = 57 o 30 , E

c) C = 37,5 o , D = 80,9 o

4.

Tnh chat 3 ng trung tuyen:


V du: Cho tam giac ABC vuong tai B, AB = 9 , BC = 12 .
Hay tnh khoang cach t trong tam G en trung iem cua cac canh
A

Giai
Ap dung nh ly Pitago trong tam giac ABC
AC =

An 9
An tiep

BC2 AB2 =

92 122

12

Ket qua :225


. Ket qua : AC = 15
1
1
1
1
GM =
BM =
AC =
15 = 2.5
3
3
2
6
AB2 BN2
1
1
GN =
AN =
92 62
3
3

Ta co: AN =

3
9
An 1
Ket qua: 3.6055
1
1
GK =
CK =
3
3
An 1

6
4.52 122

12

17

(4.1231)

Bai tap thc hanh


Cho tam giac ABC vuong tai C, CB = 16, AB = 20. Tnh khoang
cach t trong tam G en ba nh cua tam giac ABC.
S: Goi GH,GI,GJ lan lt la khoang cach t G en cac canh AC,
16
16
, GI = 4, GJ =
BC, AB. Ta tnh c: GH =
3
5
102

LP 8
AI SO
I. PHEP NHAN VA PHEP CHIA CAC A THC
1. Nhan n thc vi a thc
Tnh gia tr cua bieu thc
V du 1: Tnh gia tr cua a thc A = x(x + y) y(x + y) tai x = 2, y = 1
Giai
Trong che o
(STO)
(X) (Gan 2 cho X).
An 2
(STO)
(Y) (Gan 1 cho Y).
An 1
An
Nhap bieu thc: x(x + y) y(x + y) vao man hnh.
(X)
(X)
(Y)
An
(Y)
(X)
(Y)
Ket qua: A = 3.
V du 2: Tnh gia tr cua bieu thc x 2 2xy + 4z y 2 tai x = 1 ;
y = 1 ; z = 1.
Giai
Thay x, y, z bang A, B, C. Ta gan 1 cho A, B, C.
1
(STO)
(A) (Gan 1 cho A).
(STO)
(B) (Gan 1 cho B).
1
1
(STO)
(C) (Gan 1 cho C).

Nhap bieu thc vao man hnh


An
(A)
2
(B)
Ket qua: 2

A2 2 AB 4C B 2
(A)

(B)

(C)

Chu y: Neu bieu thc co nhieu an hn ta cung lan lt gan cho A, B,


C, . . . , M (9 an) e tnh gia tr cua bieu thc.
V du 3: Cho a thc

Px = x + ax 4 + bx 3 + cx 2 + dx + c,
biet P(1) = 1
P(2) = 4
P(3 ) = 9
P(4) = 16
P(5) = 25
a) Tnh P(6) , P(7).
b) Viet lai P(x) vi he so la cac so nguyen.
103

Giai

Ta co

a) P(x) = (x 1)(x 2)(x 3)(x 4)(x 5) + x 2

Do o P(6) = (6 1)(6 2)(6 3)(6 4)(6 5) + 6 2

= 5 4 3 2 1 + 6 2 = 156
Tng t P(7) = 769
b) Thc hien phep tnh.

P(x) = (x 1)(x 2)(x 3)(x 4)(x 5) + x 2

P(x) = x 5 15x 4 + 85x 3 224x 2 + 274x 120.


V du 4: Chng to bieu thc khong phu thuoc vao x

Q = x 2 (2x + 1) 2x(x 2 + x) + x 2 + 7
Giai

Ta co

Q = x 2 (2x + 1) 2x(x 2 + x) + x 2 + 7

= 2x 3 + x 2 2x 3 2x 2 + x 2 + 7 = 7
Vay Q = 7 (khong phu thuoc vao gia tr x)
Dung chc nang bang (Table) ta minh hoa s khong phu thuoc vao
x cua Q = x 2 (2x + 1) 2x(x 2 + x) + x 2 + 7.
An
4 (Vao mode Table)

Nhap ham f(x) = x 2 (2x + 1) 2x(x 2 + x) + x 2 + 7


An
(X)
2
(X)
1
2
(X)
(X)
(X)
(X)
7
May hoi gia tr bat au (Start?) nhap 1
May hoi gia tr ket thuc (End ?) nhap 30
May hoi moi gia tr x cach nhau bao nhieu n v (Step?) nhap 1
May hien bang sau gia tr x t 1 en 30. Vi moi gia tr x th f(x) eu
la 7.
An
, an
nhap lai gia tr au, gia tr cuoi, gia tr step khac th
thay gia tr f(x) luon la 7.
2.

Phep chia cho n thc.

Tm so d cua phep chia a thc


V du 1: Tm so d cua phep chia

3x4 5x3 4x2 2x 7


x5

Cach 1:

Ta biet phep chia

Giai
P(x)
co so d la P(a)
xa
104

at P(x) = 3x4 5x3 4x2 2x 7 th so d cua phep chia la P(5)


Ta tnh P(5) nh sau
5
(STO)
(X) (Gan 5 cho X).
An 3
(X)
4
5
(X)
4
2
(X)
7 e ghi vao man hnh
(X)
3X4 5X3 4X2 2X 7
An
thay may hien 2403.
Ket qua: P(5) = 2403 la so d cua phep chia tren.

Cach 2:
Ta co the dung s o Hooc-n e thc hien phep chia a thc
nguyen cho x a nh sau:
Ta ghi
3
5
4
2
7
5
3
53+5
20 5 4
96 5 + 2
482 5 7
= 20
= 96
= 482
= 2403

Vay
3x4 5x3 4x2 2x 7
2403
= 3x3 20x2 96x 482
x5
x5
Thc hien theo cach nay ta c cung mot luc bieu thc thng
va so d.

V du 2: Tm so d cua phep chia


x5 7x3 3x2 5x 4
x3
5

Giai

at P(x) = x 7x 3x 5x 4
Th so d cua phep chia la P(3)
Ta tnh P(3) nh sau
An
3
(STO)
(X) (Gan 3 cho X).

Ghi vao man hnh x5 7x3 3x2 5x 4


bang cach
(X)
5
7
(X)
(X)
5
(X)
4 va an
.
Ket qua: P(3) = 46 la so d cua phep chia tren

V du 3: Tnh a e x4 7x3 2x2 13x a . Chia het cho x + 6


Giai

at P(x) = x4 7x3 2x2 13x a th so d cua phep chia la P(6).


e phep chia la phep chia het th so d bang 0 tc P(6)=0.
Tnh P(6).
105

An

6
(X)

(STO)
X (Gan 6 cho X)
4
7
(X)
2
(X)
13
(X)
May hien 222 tc P(6) = a 222.
Vay e phep chia tren la phep chia het th P(6) = 0 a 222 = 0
hay a = 222.

Bai tap thc hanh


1. Tnh gia tr cua bieu thc

a) (a 2 b 2 ) + 3ab 2 4a 3 b 4 tai a = -3 ; b = 2.

S: 1697

b) (a + b c) 2 4abc + c 3 ba tai a = -2 ; b = 3 ; c = 5. S: -614


c)

a4b c3a

tai a = -1 ; b = 1 ; c = 4.

ab3 c2b
2. Tm so d cua phep chia

13
3

4x4 3x3 5x2 x 3


.
a)
x7

S: 10888

5x5 x4 3x3 x2 5x 7
.
b)
3x 5

S:

c)
II.

S:

3x4 5x3 x2 7x 3
.
x6

18526
243

S: 4893

PHAN THC AI SO
Lien phan so
V du 1: Bieu dien A ra dang phan so thng va so thap phan.
5
A=3+
4
2
5
2
4
2
5
2
3
Giai

Tnh t di len
An 3
Va an
5
4
An tiep
An tiep
5
An tiep
4
An tiep
5
233
An
May hien 4
382

2
2
2
2
3
may hien 4.609947644
106

An tiep
Ket qua A = 4

may hien

1761
382

233
1761
= 4,609947644 =
382
382

V du 2: Tnh a , b biet (a , b nguyen dng) :


329
1

1
1051 3
1
5
1
a
b
Giai
Ta co
329
1
1
1
1

1
1
1051 1051 3 64
3
3
329
9
329
329
5
64
64
1
1

1
1
3
3
1
1
5
5
64
1
7
9
9

Cach an tren may (a may ve trang thai hien th hon so an


64
1 (ab/c)) 329
1051
(May hien 3
).
329
64
An tiep
3
(may hien
).
329
9
(may hien 5
).
An tiep
64
9
An tiep
5
(May hien
).
64
1
An tiep
(May hien 7 ).
9
Ket qua: a = 7 ; b = 9
Bai tap thc hanh
1. Bieu dien B ra phan so va so thap phan
1
B=7+
1
3
1
3
1
3
4
107

43
1037
=
142
142
= 7,3(02816901408450704225352112676056338)

S: B = 7

2. Tnh a , b biet ( a , b nguyen dng )


15
1
=
1
17
1
1
a
b
S: a = 7 ; b = 2
3. Bieu dien M ra phan so

M=

1
2

1
2

1
5

HD: Tnh tng t nh tren va gan ket qua cua so hang au vao
so nh A, tnh so hang sau roi cong lai.
98
S:
.
157
III.
1.

PHNG TRNH BAC NHAT MOT AN


M au ve phng trnh
V du 1: Hay th va cho biet khang nh sau co ung khong?

x 3 3x = 2x 2 + 3x 1 x = 1.
Giai

Khang nh x 3 3x = 2x 2 + 3x 1 x = 1 la sai
V tai x = 1, hai ve cua phng trnh co gia tr khac nhau (2 va 0).
Tren may ta an nh sau:
(STO)
(X) (Gan 1 vao X)
1
(X)
3
(X)
(Ve trai la 2)
Tnh ve trai ta an:
Tnh ve phai ta an:
2
(X)
3
(X)
1
(ve phai la 0).
V du 2: Cho 2 bieu thc 3x + 2 va x 2 x + 5.
a) Hay ien gia tr cua 2 bieu thc tng ng vi cac gia tr cua x
vao bang ben di.

b) Hay cho biet phng trnh 3x + 2 = x 2 x + 5 co nghiem nao


trong cac gia tr cua x a cho.

108

x
3x + 2

x2 x + 5
Giai
a) Bang cach an
4 (Table). May hien f(x)=
Tnh gia tr cua bieu thc 3x + 2.
Ghi 3X+2 vao man hnh an 3
(X)
2
5
May hoi gia tr bat au (Start?) an
May hoi gia tr ket thuc (End?) an 5
May hoi moi gia tr cach nhau bao nhieu n v an 1
qua vao bang.

. ien ket

Tng t tnh gia tr cua bieu thc x 2 x + 5


Ta ghi X 2 X + 5 vao man hnh bang cach an
(X)
(X)
5 an
May hoi gia tr bat au (Start?) an
5
May hoi gia tr ket thuc (End?) an 5
May hoi moi gia tr cach nhau bao nhieu n v (Step ?) an 1
.
ien ket qua vao bang.
Vay co ket qua bang sau:
5
4
3
2 1
0
1
2
3
4
5

x
3x+2

13

10

11

14

17

x2 x + 5

35

25

17

11

11

17

25

b) Da vao bang ta thay tai x=1 va x=3 th 2 ve cua phng trnh


bang nhau. Vay x = 1 va x = 3 la nghiem cua phng trnh
3x + 2 = x 2 x + 5.
2.

Phng trnh a c ve dang ax + b=0


V du 1: Giai phng trnh bac nhat mot an sau
7
11
5 7
7 5
x 1 + x = 3
3 2
8 11
5
9
Giai
Viet (1) lai tren giay Ax + Bx BC = D
hay (A + B)X (D + BC) = 0
Va bien oi (2) thanh (tren giay)
x = (D + BC) (A + B)

109

(1)

5 7
Gan 1 cho A bang cach an phm nh sau :
3 2

Tng t gan

)5

7
11
cho B ;
cho C ;
5
9

(STO)

(A) .

7 5
3 8 11 cho D

Roi ghi (D+BC) (A+B) vao man hnh nh sau :

Ket qua:

(D)
(B)
(B)
an
125
20321
9
=
2244
2244

(C)

(A)

V du 2: Giai phng trnh bac nhat mot an sau


15 11
2 3
1 6
3 7
x
x
=
2 35
3 5
3 2
4 3
Giai
Viet phng trnh tren lai tren lai tren giay Ax B(x C) = D
Va bien oi (2) thanh (A B)x (D BC) = 0.
Suy ra x = (D BC) (A B).
2
3
35
(STO)
An
2 3
).
(gan A =
3 5

Tng t gan B =

1 6
3 2

,C=

3 7
4 3

,D=

roi ta ghi vao man hnh (D BC) (A B)


(D)
(B)
(C)
(B)
roi an
Ket qua:

x = 1,449181224

V du 3: Giai phng trnh


x
x
a)
4+

1
1
1
4
1
1
2
3
1
1
3
2
4
2
y
y

1
b)
1
1
1
2
1
1
3
4
5
6
110

(2)

(A).

15 11
2 3 5

(A)

Giai
4
a) at 4 + Ax = Bx suy ra x =
B A
Tnh A va B nh cac bai tren
30
17
Ta c A =
;B=
va cuoi cung tnh x
43
73
884
12556
Ket qua: x = 8
=
1459
1459
1
b) at Ay + By = 1 suy ra y =
AB
Tnh A va B nh cac bai tren
Roi tnh A + B va cuoi cung tnh y
24
Ket qua: y =
29
Bai tap thc hanh
1. Hay cho biet cac khang nh sau co ung khong?

a) (x + 2)(x 2 + 1) = 3x + 4 x = 7.

S: sai

b) x 3 + 2x 2 = 2x + 2 x = 1.

S: sai

2. Tm x , biet
1
5
21
11
x
a) 2 x 3 x =
5
7
6
5
5 8
2x
13
b)
x

7
8
6
1
5

S: x =
11 3 6
25

5 8 3 2 3
c) x

1 3 2 7

462
1237

S: x = 0,1630

7
9 x = 11 2 10
6 5
13 7
3

S: x = 9,7925

111

HNH HOC
V du: Cho hnh vuong ABCD canh bang 12. M, P lan lt la trung
1
1
BC, QD =
AD. Hay tnh chu vi va dien tch
iem AB, CD, BN =
4
4
MNPQ.

Giai
MN, NP, PQ , QM , lan lt la canh huyen cua cac tam giac MBN,
NCP, PDQ, QAM. Ap dung nh ly Pitago, taco :

Chu vi MNPQ la :

chu vi

= 2 (QM +MN)
= 2( AM2 AQ2 BM2 BN2 )
= 2( 62 92 62 32 )

An 2
Ket qua:

Chu vi MNPQ = 6 6 + 6 5 = 35,0497

Tnh dien tch :


Ta co : S MNPQ = S ABCD 2 (SAMQ SQDP )
= AB AD (AM AQ + QD DP)
= 12 12 (9 6 + 3 6)

An 12

12

Ket qua:

Dien tch S MNPQ = 72.

112

LP 9
AI SO
I.

1.

CAN BAC HAI, CAN BAC BA


Can bac hai
V du 1: So nao co can bac hai la
a) 3 ;
b) 1,2
d) 8

c) 0,2 ;

Giai
So co can bac hai la:
3 tc so o la ( 3)2. An

a)

Ket qua: 3
11
36
Ket qua: 1
=
= 1,44.
25
25

b) 1,2 tc so o la 1,2 2 .An 1.2


c) 0,2 tc so o la (0,2) 2 .An
1
Ket qua:
.
25

0.2

d) 8 tc so o la ( 8 ) 2 .An
Ket qua:

V du 2: Tnh can bac hai so hoc cua:


a) 1,21 ;
b) 1,44 ;
c) 1,69 ;
d) 1,96 ;

2.

An

1.21

Ket qua:

An

1.44

Ket qua:

An

1.69

Ket qua:

An

1.96

Ket qua:

Can bac ba
V du 1: Tm x biet:
a) 3 x = 1,7

b)

Giai:
11
10
6
5
13
10
7
5

3 (x

An tiep

c 1,1.

An tiep

c 1,2.

An tiep

c 1,3.

An tiep

c 1,4.

3) = 18

Giai

a) Ta co
tiep

x = 1,7 hay x = 1,7 3 . An 1.7

c 4,913.
113

Ket qua:

4913
1000

b) Ta co

3 (x

Ket qua:

3) = 18 hay x = 18 3 + 3. An 18

5835

V du 2: Tnh can bac ba cua


a) 1331 ;
b) 1728 ;
c) 2197 ;
d) 2744 ;
Giai
An
( )1331
. Ket qua:

1331 = 11.

An

)1728

. Ket qua:

1728 = 12.

An

)2197

. Ket qua:

2197 = 13.

An

)2744

. Ket qua:

2744 = 14.

*3. Luy tha-can so bat k.


V du 1: Tnh

a) 2 10 An 2
b) (3) 5 An
2
c)
3

Ket qua: 1024.

10
3

An

d) 4 3 An 4

Ket qua: 4 3 =

128

An
An
An
An

(
(
(
(

)4
)5
)7
)10

Ket qua:

V du 2: Tnh
a) 4 83521

c)

Ket qua: 243.

b)

d)

10

1
3

16
81

1
= 0,015625.
64

32768
1024

Giai
83521
.
32768
.
128
.
1024
.

Ket
Ket
Ket
Ket

qua:
qua:
qua:
qua:

17.
8.
2.
2.

Bai tap thc hanh


1. Tnh

a) 3 10

S: 59049

c) (7) 4

S: 2401
1
S:
5

e) 5 1

1
b)
2

d) 1,12 3
f) 3 4

114

S:

1
128

S: 1,404928
1
S:
81

2. Tnh
a) 1849
b)

2683,24

c)

729
1849

S:

d)

128 2

S: 16.

25281

e)

S:

3 867

3. Tnh
a) 3 117649

4.

S: 43
259
S:
= 51,8
5
27
43

53
17

S: 49

b)

0,032768

S: 0,32

2187

S: 3

371293
16807

S:

c)

20736

S: 12

d)

e)

262144

S: 4

f)

13
7

Tnh gia tr cua bieu thc co cha can


1
1 3
B = 3 x2 +
x 5 (6x 1)2 (x2 9)3 tai x = 4
8
16
An 4
(STO)
(X) ( Gan 4 cho X )

An tiep 3

(X)
Ket qua:

)1

(X)
B = 29.

(X)

(
9

16

1
(X)

Bai tap thc hanh

a) A =

(4x 1)(3x 5)2 (x 2 + 2x + 3) tai x = 4.

S: 10

x3 10
4x 1
1289
tai x = 3.
S:
3
2x 3
342
x 11
2
1

c) C =
tai x = 10.
2
x 1 x 6
5(x 5) x 4x 4

b) B =

S:
d) D =

3x 7 4 7x
3

x 6x

1
tai x = 2

115

27
119

S: 2,1786

II.

HAM SO BAC NHAT


ien cac gia tr cua ham so y = 3x + 2 vao bang sau:

5.3

4
3

2.17

3
7

5 7

y
Giai
Ghi vao man hnh 3(5.3) + 2 bang cach an
179
2 va an
KQ:
= 17.9
10
An
va chnh lai thanh 3(4 )+ 2 va an

An

4
va chnh lai thanh 3 + 2 va an
3

An

va chnh lai thanh 3(2.17) + 2 va an

An

3
va chnh lai thanh 3 4 + 2 va an
7

An

va chnh lai thanh 3(5 7 ) + 2 va an

5.3

KQ: 14.
KQ: 6
451
= 4.51
100
79
KQ:
7

KQ:

KQ: 2 15 7 = 37.6863
Ta c bang ket qua
x

5.3

17.9

14

4
3

2.17

43
7

5 7

4.51

79
7

37.686

V du 2: ien cac gia tr cua ham so y = 3x 2 vao bang sau:

5.3

4
3

2,17

43
7

5 7

y
Giai
Lam tng t nh v du 1, ta c ket qua

5.3

84.27

48

4
3
16
3

2,17
14.1267
116

43
7
2883
49

5 7
525

Bai tap thc hanh

1
5
, y2 =
4x , y 3 = 4x 2 + 2
2
3
Hay lap bang gia tr cua y 1 , y 2 , y 3 ng vi cac gia tr cua x la :

1. Cho cac ham so y 1 = 3x +

3,

3
1
, 1 , 0 , 2 , 3 , 4
,
2
5

19

S:
y

3
2

1
5

19

1
2

19
2

7
2

1
2

11
2

17
2

121
10

1 6 19
2

5
4x
3

41
3

23
3

17
3

5
3

19
3

31
3

227
15

15,7869

4x 2 + 2

34

14

34

1714
25

74

3x +

2.

o th cua ham so
V du 1: Cho ham so y = -5x + 4
a) Ve o th cua ham so
b) Tnh goc hp bi ng thang y = -5x + 4 va truc Ox

a. Ta co o th nh hnh ve

Giai

b. Goi goc hp bi ng thang y = -5x + 4 va truc Ox la = ABx


Xet tam giac vuong OAB , ta co
OA
4
tg OAB =
=
= 5. Tnh OAB bang cach an
4
OB
5
(Chuyen ve che o Deg (o)) an
3.
117

An

(tan1) 5

An tiep

Ket qua 78o4124.24.

Vay = 180o 78o 4124.24 = 101o1836


Ghi chu: Neu biet ng thang y = ax + b co tg = a th
= tan 1 a, cach tnh se nhanh hn .

Bai tap thc hanh


1. Tnh goc hp bi cac ng thang sau va truc Ox
1
a) y x 4 S: 180265.82
3

b) y 3x 2 S: 600
c) y = 5 2x S: 11603354.18
1
d) 2y + 3x =
S: 12304124.24
2
2. Goi giao iem cua ng thang y=2x+

vi Oy, Ox theo th t

la A, B va giao iem cua ng thang y=4x +

vi Oy, Ox theo

th t la A, C. Tnh cac goc cua tam giac ABC.

y = 4x +

y = 2x +

= 630265,82 ; C = 1040210.48.
S: A = 1203143.7 ; B

118

III.
1.

HE PHNG TRNH BAC NHAT HAI AN.


He hai phng trnh bac nhat hai an:
V du 1: Giai he phng trnh sau
13x 17y 25 0

23x 123y 103 0

Neu e cho he phng trnh khac dang chuan tac, ta luon a ve


dang chuan tac nh sau
13x 17y 25
roi bat au dung may e nhap cac he so

23x 123y 103


Giai
3 (EQN), an tiep 1 (a n X + b n Y = c n ).

An

(nhap a 1 )17

An 13
23

(nhap a 2 )

123

(nhap b 1 )

25

(nhap c 1 ).

(nhap b 2 ) 103

(nhap c 2 )

662
957
, an
y=
995
995
e thoat khoi chng trnh giai he phng trnh, ta an

Ket qua:

x=

V du 2: Giai he phng trnh


5x 2y 3 7

x 5,43y 15
Lam tng t nh tren nhap
a 1 = 5, b 1 = 2 3 , c 1 = 7

a 2 = 1, b 2 = 5.43, c 2 = 15 va an
Ket qua:

x 0.4557

y 2.6785

V du 3: Giai he phng trnh


12.241x 17.436y 25.168

23.897x 19.372y 103.618


Giai tng t nhap
a 1 = 13.241, b 1 = 17.436, c 1 = 25.168

a 2 = 23.897, b 2 = 19.372, c 2 = 103.618 va an


Ket qua:

x 1.95957

y 2.93156

V du 4: Tm m, n biet:
a) (1;2) va (4;3) thuoc ng thang mx ny = 7
b) (2;5) va (5;2) thuoc ng thang 2mx + 3ny = 6
119

1.

Giai

a) Giai he phng trnh


m 2n 7

4m 3n 7
Giai tng t nhap
va an
Ket qua:

a1 = 1, b1 = 2, c1 = 7
a2 = 4, b2 = 3, c2 = 7

m 5

n 21

b) Giai he phng trnh


4m 15n 6

10m 6n 6
Giai tng t nhap
va an
Ket qua:

a1 = 4, b1 = 15, c1 = 6
a2 = 10, b2 = 6, c2 = 6

3
7
2
7

Bai tap thc hanh


1) Hang ien thoai di ong co hai thue bao tra trc va tra sau.
Biet rang :
Gia cc thue bao tra trc la 3000 / phut
Gia cc thue bao tra sau la 1500 / phut .

Cho biet tong so thi gian trong mot thang ca hai thue bao a
thc hien cuoc goi la 3 gi 59 phut, tng ng vi so tien can
phai thanh toan theo quy nh ban au la 498000 ong. Tuy
nhien do ang trong thi gian khuyen mai nen :
Thue bao tra trc c tang 600 giay goi mien ph
Thue bao tra sau c tang 900 giay goi mien ph .
Hoi so tien thc s can phai tra cho hang ien thoai di ong cua
moi thue bao
S: Thue bao tra trc : 249000 ong
Thue bao tra sau : 196500 ong
120

2) Giai cac he phng trnh sau :


1

y x 4
a)
3
2y 3x 1

27

x 11
S:
y 35

11

4x 3y 3 0
b)
2x 1 y 4

109

x 66
S:
y 23

11

3x 5

7
c) 2y
5x 4y 5 0

x
S:
y

25
67
105
134

3) Tm m, n biet:

m
n
x
y=9
2
3
m
b) (9;7) va (4;5) thuoc ng thang
x 2ny = 11
5
S:
72
110

m 5
m 17
b)
a)
n 27
n 55

34
5

a) (3;7) va (2;3) thuoc ng thang

Ghi chu: Khi gap he vo nghiem


a1 b1 c1

a 2 b2 c2

hay he vo nh

a1 b1 c1

a 2 b2 c2

th may bao loi


2.

Giai bai toan bang cach lap he phng trnh:


V du 1: Nam nam trc tuoi me va ung gap 4 lan tuoi con. Nam
nay tuoi me bang 3 lan tuoi con cong them 5. Hoi nam nay moi
ngi bao nhieu tuoi ?
Giai
Goi so tuoi me va tuoi con lan lt la x, y; x N*, y N*, x > y > 5.
Ta co he phng trnh:
x 4y 15
x 5 4(y 5)

x 3y 5
x 3y 5
121

Giai tng t nhap


va an
Ket qua:

a1 = 1, b1 = 4, c1 = 15
a2 = 1, b2 = 3, c2 = 5

x 65

y 20

V du 2: Cho mot so 2 ch so. Neu oi cho 2 ch so cua no th c


mot so ln hn so a cho la 72.Tong cua so a cho va so mi tao
thanh bang 110.Tm so a cho.
Giai

Goi so can tm la xy , vi x, y N*; x 9, y 9.


Theo au bai, ta co he:
10y x 10x y 72
hay

10x

10y

110

Giai tng t, nhap


va an
Ket qua:

9x 9y 72

11x 11y 110

a1 = 9, b1 = 9, c1 = 72
a2 = 11, b2 = 11, c2 = 110

x 1

y 9

Vay so a cho la 19.


Bai thc hanh
1. Hai anh Quang va Hung cung gop von kinh doanh. Anh Quang
gop 18 trieu,anh Hung gop 15 trieu. Sau mot thi gian c lai 11
trieu ong.Lai c chia t le vi von gop. Em hay dung cach giai
he phng trnh e tnh tien lai ma moi anh c thng.
S: Anh Quang c 6 trieu
Anh Hung c 5 trieu.
2. Hom qua me Nam i ch mua 2 qua trng ga, 8 qua trng vt het
14600 ong. Hom nay me Nam i ch mua 4 qua trng ga, 6 qua
trung vt ch het 14200 ong ma gia trng th van nh cu. Hoi gia
mot qua trng moi loai la bao nhieu.
S: Trng ga:1300 ong/qua
Trng vt:1500 ong/qua
III.

He phng trnh bac nhat 3 an

An
e vao chng trnh giai he phng trnh bac nhat
3 an
Ta luon luon a he phng trnh ve dang

122

a1x b1y c1z d1

a2x b2y c2z d2


a x b y c z d
3
3
3
3
roi mi nhap he so lan lt vao may
V du: Giai he phng trnh sau
3x 2y 4z 7 0

x 5y z 5 0
7y 3z 3 0

Ta a ve dang :

3x 2y 4z 7

x 5y z 5 roi nhap he so
7y 3z 3

Giai
Goi chng trnh giai he phng trnh bac nhat 3 an nh sau
3 (EQN) 2
An
2
4
7
An tiep 3
1
5
1
5
0
7
3
3
110
Ket qua: x =
an tiep
. Ket qua x = 4.7826
23
21
y=
an tiep
. Ket qua y = 0.4565
An
46
95
z=
an tiep
. Ket qua z = 2.0652
An
46

e thoat khoi chng trnh giai he phng trnh, ta an


Bai tap thc hanh

3x 7y z 6 0

a) x 3y 6z 5 0

1
2 x 2y z 3 0

3z 4y x 8

b) y 3x 4z 5
2x 3 z y

76

x 25

53

S: y
25

7
z 25

18

x 5

S: y 5

26
z
5

123

3x y 2 3 z 1

c) ( 2 1)x z
7

3x 2y z 3

IV.
1.

Ham so y = ax2
Ham so y = ax2

x 4.2441

S: y 3.9158
z 1.9008

(a 0)

V du: Cho ham so y = 5x2.


Tnh cac gia tr cua y ng vi cac gia tr cua x lan lt bang 2 ; 1 ;
1
1

;0;
.
5
5

Vi x = 2. An 5
Vi x = 1. An
1
Vi x = . An
5
1
5 .

(20)
1

sa lai thanh 5

sa lai thanh 5

Vi x = 0. An
1
. An
Vi x =
5
1
5
V.

Giai
2
sa lai thanh 5

(5)
1

sa lai thanh 5

(0)
1

Phng trnh bac 2 mot an

ax 2 + bx + c = 0 (a 0)
V du 1: Giai phng trnh
73x 2 47x 25460 = 0
Goi chng trnh giai phng trnh bac 2
An
3(EQN) 3
73
(nhap a)
47
(nhap b)
25460
Ket qua: x 1 = 19

x2 =
Neu an tiep

1340
73

th x 2 = 18,3562

26
73
( ay oi ra phan so c do la so chnh phng)

Neu an tiep

th x 2 = 18
124

V du 2: Giai phng trnh

x 2 + x 3 2 5 = 0

Lam tng t nh tren vi a = 1, b =


Ket qua:

3 , c = 2 5

x1 1.4192

x2 3.1512

Ghi chu:

Khi giai phng trnh ax 2 + bx + c = 0 ma man hnh ket qua xuat


dang a + bi th ket luan la phng trnh ax 2 + bx + c = 0 vo
nghiem tren tap so thc R
(nh phng trnh x 2 + x + 1 = 0, x 2 + 1 = 0)
ac biet may se xuat ket qua ung dang bieu dien cua so vo t
neu che o
, ch khong dung so thap phan. ay chnh la
u iem cua may. Neu muon hien gia tr thap phan khi may xuat
gia tr vo ty th an
.
e thoat khoi chng trnh giai phng trnh bac 2, ta an
Bai tap thc hanh
Giai cac phng trnh bac hai sau

a) 3x 2 4x + 7 = 0

S: PTVN thc

5 13
x1
2
S:

5 13
x2
2

b) x 2 + 5x + 3 = 0

c)

2 x2 + 2 3 x

2 =0

d) (x 4) 2 + (2x + 1) 2 = 25 5x

VI.

10 6
x1
2
S:

10 6
x2
2

1 161
x1
10
S:

1 161
x2
10

Phng trnh bac 3 mot an (*)


V du 1: Giai phng trnh bac 3 sau

2x 3 + x 2 8x 4 = 0
Goi chng trnh giai phng trnh bac 3
125

1.

An

3(EQN) 4

(nhap a) 1
An 2
(nhap d)

Ket qua:

(nhap b)

(nhap c)

x1 2

x 1
2
2
x 2
3

V du 2: Giai phng trnh bac 3 sau


2 x 3 5x 2 +

3
2

15
=0
2

Lam tng t nh tren, ta thay phng trnh a cho ch co mot


nghiem thc la x = 3.5355 (hai nghiem con lai eu la nghiem
phc (co ch i), khong nhan).
e thoat khoi chng trnh giai phng trnh bac 3, ta an
Giai cac phng trnh bac 3 sau (ch tm cac nghiem thc)
a) x 3 + x 2 3x + 3 = 0
b)

3 x3 + x2

S: 2,5987

3
1
x
=0
2
2

c) 3x 3 + 2x 2 x + 14 = 0
d) x 3

x1 0,7071

S: x2 0,7071

x3 0,5774
S: 2

15 2
27
x + 18x
=0
2
2

126

x1 2
S:
x 3
2,3

HNH HOC
I.

T so lng giac cua goc nhon


( cap 2, ta cho man hnh hien D ( o))
V du 1: Tnh

a) sin36 o
b) tg78 o
c) cotg62 o
a) An
b) An
c) An 1

Giai

36
78

S: 0.5878
S: 4.7046
S: 0.5317

62

V du 2: Tnh

a) cos43 o 27'43
b) sin71 o 5214
c) tg69 o 057
a) An
b) An
c) An

43
71
69

27
52
0

Giai
43
14
57

S : 0.7258
S : 0.9504
S : 2.6072

V du 3: Tm goc nhon X bang o, phut, giay biet


a) sinX = 0.5
b) cosX = 0.3561
3
c) tgX =
4
d) cotgX = 5
Giai
1

S: 30 o

a) An

(sin

) 0.5

b) An

(cos 1 )0.3561

S: 69 o 821

c) An

(tan 1 ) 3

S: 36 o 5212

d) An

(tan 1 ) 1

4
5

S: 24 o 541

V du 4: Cho tam giac ABC vuong tai A, canh AB = 3.26 cm, goc
= 51 o 26. Tnh AC, BC va ng cao AH.
B

127

Giai

51026
A

3,26 cm

AC = AB tgB = 3.26 tan56 o 26 = 4.0886 cm


AB
AB
= cosB BC =
= 5.2292 cm
BC
cosB
AH = AB sinB = 2.5489
BC 2 = AB 2 + AC 2
1
1
1
AH t cong thc
=
+
AH2
AB2
AC2
hay t cong thc AH BC = AB AC)

(Co the tnh BC t cong thc

V du 5: Cho tam giac ABC vuong tai A, canh AB = 5cm ;


AC = 12cm. Tnh BC, goc B, goc C.
Giai

BC

= AB
AC
tgB =
AB

An
An tiep 90

(tan 1 )12

+ AC

= 13cm

va an

= 67 o 2248
S: B

S: C = 22 o 3712

128

II.

Tnh gia tr cua bieu thc


V du:

A = 7 cos 2 60 o + 2sin 2 45 o +

1 2
tg 30 o
2

Giai

a) An
An 7
1
2
95
S:
12

3(Deg)
60
30

45

Bai tap thc hanh


Tnh gia tr cua bieu thc

B=

2 3 3 sin3 90o cot g3 30o cos2 45o


tg4 60o sin2 30o cos3 60o

1
sin2 40o cos2 20o
o
C=
cotg55 +
3
tg3108o

S:

80
289

S: 0,2209

V du 1: Cho tam giac ABC vuong tai A, canh AB = 2AC. Tren


canh huyen BC, lay iem I vi CI = CA, tren canh AB lay iem K vi
BK = BI. ng tron tam K, ban knh KB cat trung trc cua KA tai
iem M.

Tnh goc MBA


Giai

129

at AB = 2AC = 2a th BK = BI = a( 5 1) va KA = a(3

5)

Goi L la trung iem cua KA, tam giac LKM vuong tai L cho ta
a
(3 5)
KL
3 5
2
=
cos( MKL ) j=
=
KM
2( 5 1)
a( 5 1)
An

3 (Deg)
(cos-1)
va an
1
May hien 72, ta co

MKL = 72 o = 2 MBA

MBA = 36 o

Ghi chu: Bai toan nay co the dung e ve goc 36 o bang thc dai va
compa ngha la ve ngu giac eu noi tiep trong ng tron bang
thc va compa.
V du 2: Tnh khoang cach gia hai nh khong lien tiep cua mot
ngoi sao 5 canh noi tiep trong ng tron ban knh R = 5.712cm.
Giai

AC = 2Rcos18 o = 10.8649cm
A

E
O

A
130

V du 3: Tnh dien tch hnh tron noi tiep tam giac eu canh
a = 12.46cm.
Giai
1
3
Ban knh r cua ng tron phai tm la r = a
3 2

Va dien tch phai tm la S = a 2 = 40.6448cm 2


Cach an may
3
6
12.46
(STO)
Gan cho A:

Va ghi tiep A 2 va an
Ket qua:
III.
1.

S = 40.6448cm 2

Hnh tru - Hnh non Hnh Cau:


Hnh tru :
V du 1: Mot mieng ton hnh ch nhat co chieu dai 40cm chieu
ngang 10cm c cuon lai thanh be mat xung quanh cua mot hnh
tru cao 10cm. Tnh the tch hnh tru ay.
Giai
Goi ban knh ay hnh tru la R . Ta co
20
2 R = 40
hay R =

10
20
= 1273.2395cm 3
10 = 20 2
The tch V = R h =

An 20

10

( ) va an

V du 2: Mot hnh tru ngoai tiep mot hnh hop ng ay vuong canh
25.7cm, cao 47.3cm .Tnh dien tch xung quanh cua hnh tru va the
tch phan khong gian gii han gia hnh tru va hnh hop.
Giai
Goi canh ay hnh hop la a, chieu cao h, ban knh hnh tru la R
2
Ta co R = a
2
Dien tch xung quanh S cua hnh tru la
a 2
3
S = 2 Rh = 2
h = 25,7 47,3 2 = 5400,8129cm
2

)
(Ghi vao man hnh 25.7 47.3 2 va an
The tch phai tnh la

V t V h = R 2 h a 2 h = a 2 h 1
2
131

= 25,7 2 47,3(0,5 1)
= 17832,349cm 3
47.3
0.5

An 25.7
va an
2.

()

Hnh non Hnh Cau


V du 1: Mot hnh tron ban knh R = 21.3cm c cat bo mot phan t
e xep thanh be mat xung quanh cua mot hnh non. Tnh:
a) Dien tch mat ay cua hnh non.
b) Goc nh cua hnh non.
c) The tch cua hnh non.
Giai
a) Goi r la ban knh ay, ta co
3
2r =
2R
r = 0,75R
4
= 0,75 21,3 = 15,975cm
Do o, dien tch ay

S = r 2 = 15,975 2 = 801,7364cm 2
()
15,975
An
b) Goi goc nh la 2 th
r
= 0,75
sin =
R
Tnh 2 , bang cach an
(sin-1) 0,75 va an

2
Ket qua:
c) The tch

2 = 97 o 10'51"

1
r2h
3
1
=
15.975 2
3

V =

An 1
15.975

21,32 15,9752 = 3765,121cm 3

15.975

21.3

va an

V du 2: Mot hnh non co chieu cao la 17.5cm, ban knh ay 21.3cm


c ay len mot hnh cau sao cho mat cau tiep xuc vi mat xung
quanh va vi mat ay cua hnh non. Tnh dien tch mat cau va the
tch hnh cau.

132

Giai

17.5
ABH
r = 21,3tan
21.3
2
Tnh r = E bang cach ghi vao man hnh nh sau
0.5
17.5
21.3
21,3
(E)

tan ABH =

Dien tch S = 4 E 2 = 731,1621cm 2


4
E 2 = 1859,0638cm 3
The tch V =
3

133

(STO)

Chu trach nhiem xuat ban :


Chu tch HQT kiem Tong Giam oc NGO TRAN AI
Pho Tong Giam oc kiem Tong bien tap NGUYEN QUY THAO

Bien tap noi dung :


NGUYEN ANG TR TN

Bien tap k thuat :


CTY CP CNTT TR C
Trnh bay ba :
HA TUE HNG
Sa ban in :
HOAI TN TR TN
Che ban tai :
CTY CP CNTT TR C

Hng dan s dung va giai toan tren may tnh Casio fx-500 Vn plus
dung cho lp 6-7-8-9. Ma so : ..................................
In .................. ban (Q .................) kho 14,520,5cm tai ........................
So in. So KKH xuat ban ..............................In xong va nop lu chieu
ngay .... thang .... nam 2009
134

135

You might also like